Módulo 5 Med. Interna Flashcards
A 26-year-old man comes to the physician because of sore throat, fever, and malaise for 1 week, and a diffuse skin rash for 1 day. The skin rash developed after the patient took ampicillin. Examination reveals pharyngitis and tonsillitis, cervical lymphadenopathy, and splenomegaly. Laboratory studies show:
- Hematocrit 40%
- Leukocyte count 4800/mm3
- Segmented neutrophils 45%
- Lymphocytes 40%
- Platelet count 76,000/mm3
- Alanine aminotransferase 80 U/L
- Aspartate aminotransferase 70 U/L
- Bilirubin, total 1.2 mg/dL
A peripheral blood smear show numerous atypical large lymphocytes with vacuolated cytoplasm. A heterophil antibody test is positive. Which of the following is the most likely diagnosis?
(A) Acute cytomegalovirus (CMV) disease
(B) Acute lymphocytic leukemia
(C) Drug-induced thrombocytopenia
(D) Infectious mononucleosis
(E) Streptococcal pharyngitis with leukemoid reaction
Respuesta: D
The correct answer is D. Characteristics of infectious mononucleosis include signs and symptoms similar to influenza, with lymphadenopathy (especially in the cervical chain) and splenomegaly. Skin rash is infrequent, but ampicillin administration is followed by a diffuse maculopapular rash in 90% of cases. Atypical lymphocytes are easily identified on blood smears. Granulocytopenia is present initially, and thrombocytopenia develops frequently. IgM antibodies to Epstein-Barr virus appear during the acute phase. The Monospot test is based on heterophil (sheep cell agglutination) antibody tests and becomes positive before the 4th week after the onset of the disease.
Acute cytomegalovirus (CMV) disease (choice A) may be indistinguishable in its clinical and laboratory manifestations from infectious mononucleosis, but the heterophil antibody test is negative in CMV infection. Most CMV infections, however, remain asymptomatic in immunocompetent hosts, producing severe disseminated infections in immunocompromised patients.
Acute lymphocytic leukemia (choice B) is a disease of children (peak between 3 and 7 years) and is characterized by pancytopenia and the presence of circulating blasts.
Drug-induced thrombocytopenia (choice C) is not associated with other hematologic abnormalities or systemic illness. Currently, most cases are due to heparin. Other common drugs that may cause thrombocytopenia include sulfonamides, thiazides, and cimetidine.
Streptococcal pharyngitis with a leukemoid reaction (choice E) would be associated with marked neutrophilic leukocytosis with the presence of numerous circulating granulocytic precursors. Leukemoid reactions may give rise to leukocyte counts up to 50,000/mm3 , but are not associated with the presence of blasts or thrombocytopeni
A 65-year-old man presents with a productive cough, fever, chills, and shortness of breath for 3 days. On physical examination, his temperature is 38.9 C (102 F), pulse is 96/min, and respirations are 28/min. There are decreased breath sounds and dullness to percussion over the left lower field. Laboratory findings are remarkable for a white cell count of 21,000/mm3 with 20% bands. A chest x-ray film demonstrates the presence of a large left pleural effusion. A thoracentesis is performed. Which of the following is the proper position for insertion of a thoracentesis needle within the sixth intercostal space?
(A) Inferior edge of the sixth rib at the midclavicular line
(B) Inferior edge of the sixth rib between the tip of the scapula and the posterior axillary line
(C) Superior edge of the seventh rib at the midclavicular line
(D) Superior edge of the seventh rib between the tip of the scapula and the posterior axillary line
Respuesta: D
The correct answer is D. The proper placement of the needle is at the superior edge of the seventh rib between the tip of the scapula and the posterior axillary line (rather than the midclavicular line, choice C). The superior border of the rib is used to avoid the neurovascular bundle, which runs along the inferior edge of each rib. Before the tap is done, a lateral decubitus film should be performed to confirm that the effusion is free flowing. The lungs should be percussed, and the needle should be placed below the point where fluid is detected. This usually occurs at the seventh rib. At least 200 mL of fluid should be present to be successfully tapped (otherwise it should be done with ultrasound guidance).
Insertion at the inferior edge (choices A and B) can result in damage to the neurovascular bundle.
A 66-year-old man comes to the physician for his annual health maintenance examination. He feels fine, and his vital signs are normal. Examination is remarkable only for enlarged lymph nodes in the cervical and sup-raclavicular regions. Liver and spleen are not palpable. Blood studies reveal:
- Hematocrit 45%
- Hemoglobin 13.5 g/dL
- Platelet count 230,000/mm3
- Leukocyte count 28,000/mm3
- Lymphocytes 80%
- Neutrophils 15%
- Monocytes 5%
Peripheral blood smear shows a large number of mature lymphocytes. Bone marrow biopsy demonstrates diffuse infiltration by mature-looking lymphocytes. Which of the following is the most appropriate treatment at this time?
(A) No treatment
(B) Chlorambucil
(C) Fludarabine
(D) Prednisone
(E) Bone marrow transplantation
Respuesta: A
The correct answer is A. The diagnosis is chronic lymphocytic leukemia (CLL). CLL is a neoplastic disorder of B lymphocytes, characterized by marked peripheral lymphocytosis. Circulating lymphocytes are extremely similar to normal lymphocytes and tend to accumulate progressively in the marrow and blood because of inactivation of the apoptosis-inhibiting Bcl-2 gene. The manifestations are due to increasing immunosuppression, bone marrow replacement, and organ infiltration. Lymphocytosis due to CLL is often incidentally discovered in otherwise healthy older people. Lymphadenopathy, however, is frequently present. CLL follows an indolent course, and aggressive chemotherapy seems to have little impact on survival. In patients who present with only lymphocytosis and lymphadenopathy (stage I according to the Rai system), no treatment is necessary.
Chlorambucil (choice B) is the standard initial treatment for symptomatic CLL, i.e., patients with progressive fatigue and organomegaly (stage II), severe anemia (stage III), and thrombocytopenia (stage IV). Chlorambucil is well tolerated.
Fludarabine (choice C) is usually used as a second-line treatment for patients who do not respond any longer to chlorambucil. It is associated with significant immunosuppression.
Prednisone (choice D) is useful for certain autoimmune-mediated manifestations of CLL, namely autoimmune hemolytic anemia or thrombocytopenia.
Bone marrow transplantation (choice E) is used only for the rare young patient who presents with an aggressive form of CLL. It is not a treatment option for most CLL patients. These are elderly persons who most often will die of causes unrelated to CLL
An 18-year-old Caucasian woman presents to her physician with complaints of excessive thirst over the past several months. Dipstick urinalysis demonstrates 4+ glucose in the urine. Blood chemistries demonstrate glucose of 420 mg/dL. In Caucasian patients, the condition affecting this woman is most strongly associated with which of the following HLA types?
(A) DR1 and DR2
(B) DR1 and DR3
(C) DR2 and DR3
(D) DR2 and DR4
(E) DR3 and DR4
Respuesta: E
The correct answer is E. This is a typical presentation of type 1 diabetes mellitus. Patients are typically diagnosed in childhood or adolescence, and this form of diabetes is the most prevalent type in patients younger than 30. Type 1 diabetics are particularly prone to develop diabetic ketoacidosis, as they produce little or no insulin. In Caucasian patients, there is a strong association between type 1 diabetes mellitus and the specific HLA phenotypes HLA-DR3, HLA-DR4, and the heterozygote form HLA-DR3/HLA-DR4. These patients tend to have detectable serum islet cell cytoplasmic antibodies or antibodies to glutamic acid decarboxylase and to insulin. It is thought that their diabetes mellitus is the result of an immunemediated selective destruction of the islet beta cells that usually secrete insulin. Pancreatic biopsies in these patients (usually done in research rather than clinical settings) show a dense lymphocytic infiltrate in pancreatic islets, with T and B lymphocytes, macrophages, and a loss of most beta cells.
HLA-DR1 and HLA-DR2 (choices A to D) are not associated with an increased incidence of type 1 diabetes and are actually relatively “protective” HLA types, since most type 1 diabetics instead have HLA-DR3 or HLA-DR4
A 45 year-old man undergoes a routine physical examination with screening blood studies. Physical examination is notable for an increased liver diameter; the liver edge is palpable and without irregularities. Blood studies show elevated liver enzymes. The clinician suspects alcoholic hepatitis. Which of the following findings would tend to support this diagnosis?
(A) Alanine aminotransferase = 2000 U/L
(B) Aspartate aminotransferase (AST)/alanine aminotransferase (ALT) ratio = 2.5
(C) Gamma-glutamyl transferase (GGT) = 20 U/L (norm ≤65 U/L)
(D) Mean corpuscular volume (MCV) = 65 μm3
(E) Platelet count = 600,000/mm3
Respuesta: B
The correct answer is B. The diagnosis of alcoholic hepatitis has enough social implications that it is important to be reasonably sure it is correct before suggesting it. A helpful rule of thumb in making the diagnosis is that the ratio of serum aspartate aminotransferase (AST) to serum alanine aminotransferase (ALT) is usually greater than 2 in alcoholic hepatitis. Other indices are affected by alcoholic hepatitis but are not as specific for the disease.
Very high ALT levels (choice A) are more characteristic of viral hepatitis. In alcoholic hepatitis, elevation to around 250 U/L is more typical.
The gamma-glutamyl transferase (GGT) level (choice C) provides another helpful clue to alcoholic hepatitis, in that it is often markedly elevated in these patients. Normal levels for males are less than or equal to 65 U/L; for females, less than or equal to 45 U/L.
The erythrocyte mean corpuscular volume (MCV) (choice D) is frequently elevated and can be used as a marker of alcoholic hepatitis because it gradually returns to normal with drinking cessation.
Platelet count (choice E) is often decreased as either a direct toxic effect or secondary to hypersplenism.
A 38-year-old man comes to his physician because of irregular jerky movements of his upper extremities for the past 3 months. The man’s wife says that, over the past year, he has been acting irritable, moody, and restless. He stopped playing chess because he was unable to concentrate and sit quietly for long periods of time. He has no siblings. His father died at the age of 60 years after developing a dementing disorder in his forties. His grandmother died in a mental institution. During the examination, the patient displays erratic movements of his arms and fingers, which he tries to suppress or disguise. Mental status examination reveals difficulty in concentration and a mildly depressed mood, but intact short-term memory. No focal neurologic deficits are present. Which of the following is the most likely diagnosis?
(A) Creutzfeldt-Jacob disease
(B) Gilles de la Tourette syndrome
(C) Huntington disease
(D) Sydenham chorea
(E) Tardive dyskinesia
Respuesta: C
The correct answer is C. The clinical manifestations and family history are consistent with Huntington disease, which is due to an autosomal dominant mutation of a gene on chromosome 4. The mutation consists of an unstable expansion of a CAG trinucleotide repeat in a gene encoding a novel protein named huntingtin. The age of clinical onset is commonly between 30 and 50 years, but may be as early as 5 y. Behavioral abnormalities often precede the characteristic choreiform movements. The patient may experience irritability, restlessness, and difficulty in concentration. Dementia subsequently develops.
Creutzfeldt-Jacob disease (choice A) manifests with a rapidly progressive dementia and mental status changes associated with myoclonic movements. A family history is usually absent.
Gilles de la Tourette syndrome (choice B) has an early clinical onset, commonly between 2 and 15 years of age. The manifestations include motor tics (sniffing, blinking, spitting, head and shoulder movements) or phonic tics (grunts, coughs, verbal sounds, coprolalia).
Sydenham chorea (choice D) accompanies rheumatic disease and is one of the major Jones criteria for the diagnosis of this condition.
Tardive dyskinesia (choice E) is a late complication of antipsychotic drugs that block dopamine D2 receptors. It most commonly involves the lower face, manifesting with persistent chewing movements and intermittent protrusion of the tongue.
A 24-year-old woman presents with a chief complaint of “not feeling well” for the past several days. She describes generalized weakness and is at times confused and fatigued, but she denies fever, nausea, vomiting, or abdominal pain. She has been drinking a lot of fluids and urinates every 1-2 hours during the day. She also gets up once or twice at night to urinate. She has a history of major depression, for which she is regularly seeing a psychiatrist. Examination shows she is well hydrated. Her temperature is 37.1 C (98.8 F), blood pressure is 120/80 mm Hg, pulse is 70/min, and respirations are 18/min. Her heart is regular, with a grade 2/6 systolic murmur best heard along the left sternal boarder. Which of the following sets of laboratory findings is most compatible with the diagnosis?
(A) Hypernatremia and high urine sodium
(B) Hyponatremia and high urine sodium
(C) Hypernatremia and low urine sodium
(D) Hyponatremia and low urine sodium
(E) Hypernatremia and normal urine sodium
Respuesta: D
The correct answer is D. The combination of low serum sodium and low urine sodium indicates an increase in total body water due to primary polydipsia, leading to dilutional hyponatremia. This is caused by suppression of arginine vasopressin (AVP) secretion. Deficiency of AVP produces central diabetes insipidus. Resistance to AVP at the kidney level is termed nephrogenic diabetes insipidus.
An elderly Asian man comes to the emergency department because of the rapid onset of severe pain and blurred vision in his right eye. He also reports seeing halos around lights. On examination, the eye is red with a fixed and dilated pupil. He is taking imipramine for treatment of depression. Which of the following diagnostic procedures should be performed at this time?
(A) Direct ophthalmoscopy
(B) MRI of the head
(C) Slit-lamp examination
(D) Tonometry
(E) Visual field assessment
Respuesta: D
The correct answer is D. The symptomatology of red eye, extreme pain, and blurred vision with halos around light is characteristic of narrow-angle glaucoma. This condition is most frequent in Asians and generally in individuals with narrow angles of the anterior chamber (1% of all people older than 35). It is an emergency to be promptly treated with drugs that lower intraocular pressure, such as acetazolamide or osmotic diuretics (glycerol or mannitol). Tonometric measurement of intraocular pressure confirms the diagnosis, but the affected eye even feels hard on palpation.
Direct ophthalmoscopy (choice A) to evaluate the retina, along with tonometry and visual field testing (choice E), is the mainstay for the diagnosis and management of open-angle glaucoma. Direct ophthalmoscopy allows examination of the optic disk for glaucomatous cupping. Visual field assessment documents loss of peripheral vision in open-angle glaucoma, which may result in “tunnel vision.”
MRI of the head (choice B) would have no diagnostic value in this case.
Slit-lamp examination (choice C) is particularly useful in the evaluation of lens opacities (cataract) or other pathology in the anterior chamber.
A 19-year old HIV-positive female prostitute presents to the emergency department with complaints of high fever and shaking chills for the past 48 hours. She complains of difficulty breathing and a cough productive of sputum. On chest auscultation, coarse crackles can be heard, more prominently over the right lung fields. A chest x-ray film shows patchy consolidation of right middle and adjacent upper lobe areas. Her CD4 count was 150 2 months ago. Which of the following is the most likely pathogen?
(A) Isospora belli
(B) Mycobacterium avium-intracellulare
(C) Mycobacterium tuberculosis
(D) Pneumocystis carinii
(E) Streptococcus pneumoniae
Respuesta: E
The correct answer is E. The most common form of pneumonia in AIDS patients is bacterial pneumonia. The pneumococcus is the bacterial pathogen most likely to cause the acute onset of severe symptomatic pneumonia; systemic symptoms include fever, chills, rigors, and gastrointestinal complaints. Given this patient’s presentation, Streptococcus pneumoniae is the most likely pathogen.
Isospora belli (choice A) is not a known pulmonary pathogen. It is known, however, to cause diarrhea in HIV-positive individuals.
Mycobacterium avium-intracellulare (choice B) causes a wasting systemic disease in HIV-positive patients and not a pneumonia-like picture. It is known, however, to colonize the airways of patients with chronic obstructive pulmonary disease.
Tuberculosis (TB), caused by Mycobacterium tuberculosis (choice C), usually presents as a chronic or subacute illness with cough, fever, drenching night sweats, malaise, and weight loss. Even in HIV-positive patients, TB rarely causes dyspnea. When it does, there is usually multilobar involvement or disseminated miliary TB.
Pneumocystis carinii (choice D) causes pneumonia (PCP) in patients with deficient cell-mediated immunity. It is the most common opportunistic infection in HIV-positive patients. It develops over weeks, with a dry cough, fever, and progressively worsening dyspnea, especially with exertion.
A 40-year-old woman presents with diffuse generalized edema and ascites. Laboratory studies show marked hypoalbuminemia, elevated serum creatinine, and severe proteinuria (10 g/day) without hematuria. A renal biopsy shows changes consistent with membranous glomerulopathy. Which of the following laboratory findings is most likely to be associated with this clinical condition?
(A) CD4 cell count less than 200 cells/mL
(B) Circulating C3 nephritic factor
(C) Elevated antistreptolysin levels
(D) Elevated serum IgA antibodies
(E) Positive antineutrophil cytoplasmic antibodies (ANCA)
(F) Positive antinuclear antibody test
(G) Positive serology for hepatitis B virus
Respuesta: G
The correct answer is G. It is clear that this woman has severe nephrotic syndrome. Membranous glomerulopathy is the most common cause of nephrotic syndrome in adults (minimal change disease shares this distinction in children). This disease is immune complex-mediated, and subepithelial immune deposits can be demonstrated by ultrastructural studies. Although often idiopathic, many cases are associated with some underlying condition, infectious or noninfectious. Among infectious causes, hepatitis B is the most common (syphilis is second in frequency).
CD4 cell count less than 200 cells/mL (choice A) is a defining feature of AIDS. The most common renal complication of AIDS is focal segmental glomerulosclerosis.
Circulating C3 nephritic factor (choice B) refers to a circulating IgG that binds to C3 convertase, stabilizing this factor and promoting activation of the alternative pathway of complement. This pathogenetic mechanism has been observed in type 2 membranoproliferative glomerulonephritis, which manifests with nephritic or combined nephritic/nephrotic syndrome.
Elevated antistreptolysin levels (choice C) would be present in a patient with recent streptococcal infection, which may lead to acute postinfectious glomerulonephritis. This glomerulopathy manifests with nephritic syndrome. Proliferative glomerulonephritis would be seen in a renal biopsy.
Elevated serum IgA antibodies (choice D) are associated with 50% of cases of Berger disease, the most common form of glomerulonephritis. Berger disease manifests with recurrent hematuria and/or proteinuria, often in concomitance with an upper respiratory flulike syndrome or intestinal infection. Mesangial expansion with deposition of IgA is found on biopsy.
Positive antineutrophil cytoplasmic antibodies (ANCA) (choice E) are characteristic of pauci-immune glomerulonephritides, which include Wegener granulomatosis, Churg-Strauss syndrome, and microscopic polyangiitis. All these conditions are associated with sites of involvement other than kidneys, e.g., lungs and skin.
A positive antinuclear antibody test (choice F) would be found in a patient with SLE or other collagen vascular diseases. The incidence of glomerulopathy is particularly high in SLE, in which renal impairment represents an important cause of morbidity and mortality
A 45-year-old man comes to the physician because his “face and voice have changed.” The patient came to this realization after meeting a nephew, who had not seen him for 2 years and could hardly recognize him. He also reports that he has had persistent joint pains in the past 6 months. His blood pressure is 140/90 mm Hg, but he says he has never had values over 120/80 mm Hg on previous health maintenance examinations. Physical examination reveals coarse facial features, a large tongue, and thick fingers. His handshake is moist and doughy, and his voice deep. Which of the following is the most appropriate next step in diagnosis?
(A) CT scans of the head
(B) MRI of the head
(C) Measurement of baseline growth hormone levels
(D) Measurement of growth hormone levels following glucose suppression test
(E) Measurement of TSH levels
Respuesta: D
The correct answer is D. The clinical presentation is consistent with acromegaly, due to overproduction of growth hormone (GH). This syndrome manifests when excessive production of GH occurs in adulthood after closure of the epiphyseal growth plates. Consequently, hypertrophy of acral bones and skeletal muscle results, leading to enlargement of the tongue, hands, feet, and craniofacial skeleton. Hypertrophy of pharyngeal and laryngeal tissues makes the voice deeper. Changes in facial morphology and voice are often not recognized by the patient or friends and relatives since they develop slowly, but are commonly recognized by people who have not seen the patient for many years. Often, the patient realizes that his hat, ring, or shoes do not fit any more. Secondary diabetes mellitus, arthritis, hyperhidrosis (with moist and doughy handshake), cardiomegaly, and hypertension are frequent manifestations. GH-producing pituitary adenomas are the usual cause. The diagnosis, once suspected, is confirmed by a glucose suppression test: serum levels of GH are assessed in a blood sample drawn after an overnight fast and following a challenge with 100 g oral glucose. A GH concentration higher than 2 ng/mL in men or 5 ng/mL in women is considered positive.
CT scans of the head (choice A) and MRI (choice B) of the head are used to reveal a pituitary adenoma following a positive GHsuppression glucose test. MRI is the method of choice for pituitary lesions.
Measurement of baseline growth hormone levels (choice C) is inadequate. Some patients may have normal baseline GH levels, but exercise, stress, hepatic or renal diseases, and a number of drugs may produce abnormally high levels.
Measurement of TSH levels (choice E) would be appropriate in case of suspected hyper- or hypothyroidism.
A 30-year-old woman goes to her physician complaining of a whitish discharge from her nipples for the past 3 months. She denies headaches or visual problems. She notes that she often feels tired but attributes it to the fact that she has to take care of her 18-monthold daughter. She has not had a menstrual period since her delivery and has not nursed her baby for the past 5 months. She is not taking any medications. Her physical examination is unremarkable. She does not have any visual field defects. Laboratory results show a prolactin level of 200. Which of the following is the most appropriate next step in diagnosis?
(A) Chest x-ray film
(B) CT scan of the brain
(C) Measurement of thyroid hormone levels
(D) No additional investigation is warranted
(E) Pregnancy test
Respuesta: E
The correct answer is E. The most common cause of secondary amenorrhea is pregnancy. The woman is of child-bearing age, so the first possibility is that she is pregnant (consistent with a prolactin level of 200). Also, although prolactinoma is usually associated with prolactin levels greater than 200, it might be present in this patient.
A CT scan (choice B) could be performed to rule this out, but first the physician should rule out the possibility of pregnancy.
A chest x-ray film (choice A) is unlikely to aid in the diagnosis, and, again, pregnancy should be ruled out first.
Hypothyroidism can increase prolactin le.vels moderately but would not be likely to produce such a marked elevation as that observed in this patient; therefore, measurement of thyroid hormone levels (choice C) is not necessary at this time. Some of the symptoms associated with hypothyroidism are fatigue, cold intolerance, slow movements, and weight gain.
A prolactin level of 200 is not normal and warrants further investigation (compare with choice D).
A 60-year-old man comes to his physician because of right flank pain. He has been smoking two packs of cigarettes daily for 30 years but has never had any renal diseases. His temperature is 37 C (98.6 F), blood pressure is 136/85 mm Hg, pulse is 70/min, and respirations are 14/min. Examination reveals tenderness on percussion of the right costovertebral angle and a deep abdominal mass in the right upper quadrant. Urine dipstick test shows microscopic hematuria. Which of the following is the most likely diagnosis?
(A) Adult polycystic kidney disease
(B) Angiomyolipoma
(C) Hydronephrosis
(D) Renal cell carcinoma
(E) Simple renal cyst
Respuesta: D
The correct answer is D. The triad of flank pain, hematuria, and abdominal mass is highly suggestive of renal cell carcinoma, although it represents the clinical presentation in only 20% of cases. Ultrasonography and CT scan studies are used for further diagnostic investigations. Most commonly, renal cell carcinoma presents with an isolated finding, such as hematuria (microscopic or macroscopic), weight loss, or flank pain. Note the history of cigarette smoking, which is a recognized risk factor for this type of cancer.
Adult polycystic kidney disease (choice A) manifests with hypertension, microscopic hematuria, and bilateral kidney enlargement.
Angiomyolipoma (choice B) is a rare tumor composed of a mixture of adipose tissue, blood vessels, and smooth muscle. It is a hamartomatous lesion, most often found in association with tuberous sclerosis.
Hydronephrosis (choice C) may result in a palpable renal mass, but is usually preceded by a history of urologic problems resulting from obstructive uropathy. Among these, the most frequent are renal stones, prostatic hyperplasia, vesicoureteral reflux, and anomalies in the ureteropelvic junction.
Simple renal cyst (choice E) is probably the most common lesion found in the kidneys. Rarely, it presents as a palpable mass. It is usually an incidental finding in the course of investigations undertaken for other reasons. Once a renal lesion is discovered, the problem of whether it is a common renal cyst or a carcinoma must be addressed with further investigation. Ultrasonography is the method of choice
A 40-year-old woman comes to the physician because of fever and chills, jaundice, and right upper abdominal pain radiating to the shoulder for 24 hours. At present, the patient’s temperature is 39 C (102 F), blood pressure is 100/60 mm Hg, pulse is 110/min, and respirations are 20/min. She is admitted for further diagnostic evaluation. Serum chemistry studies show:
- ALT 100 U/L
- AST 80 U/L
- Alkaline phosphatase 800 U/L
- Bilirubin Total 4.5 mg/dL
- Direct 3.5 mg/dL
- Prothrombin time 12 sec
- Amylase 200 U/L
White blood cell count is 12,000/mm3 , with 70% neutrophils. Which of the following is the most likely diagnosis?
(A) Acute cholecystitis
(B) Acute hepatitis
(C) Acute pancreatitis
(D) Choledocholithiasis with cholangitis
(E) Cystic duct syndrome
Respuesta: D
The correct answer is D. The complex of fever, right upper abdominal pain, and jaundice is referred to as Charcot triad, which is diagnostic of acute cholangitis. This usually results from a small gallstone impacted within the common bile duct. Blockage of the common bile duct results in cholestatic jaundice: hence, elevated bilirubin (mostly direct) and high serum alkaline phosphatase. Gram-negative enteric bacteria then penetrate into the biliary ducts and cause ascending cholangitis, with resultant fever and neutrophilic leukocytosis. The pain is due to acute distention of the gallbladder. This condition must be urgently treated with cholecystectomy.
Acute cholecystitis (choice A) is associated with gallstones usually impacted within the cystic duct. Colicky pain is not associated with fever, but jaundice occurs in a minority of cases. This condition (as any other form of symptomatic cholelithiasis) must be treated with cholecystectomy.
Acute hepatitis (choice B) manifests with mild right upper abdominal pain, nausea, anorexia, and low-grade fever. Serum levels of AST and ALT are markedly elevated, helping to differentiate this condition from cholangitis.
Acute pancreatitis (choice C) manifests with extremely intense deep epigastric pain, usually radiating to the back. High serum levels of amylase and lipase support a diagnosis of acute pancreatitis. However, mild elevation of amylase may be seen with cholangitis, and mild jaundice may be present in acute pancreatitis.
The designation of cystic duct syndrome (choice E) is given to clinical conditions characterized by dyspeptic symptoms (e.g., upper abdominal discomfort, nausea, bloating, and flatulence) that manifest after meals and are caused by biliary dysfunction. There are two situations in which cystic duct syndromes may develop: precholecystectomy, resulting from obstruction of the cystic duct by fibrosis or kinking, and post-cholecystectomy, due to incorrect diagnosis, neuroma of the cystic duct stump, foreign body granuloma, and common bile duct anomalies, for example.
A 30-year-old, dark-skinned man of racially mixed descent consults a physician because “his eyes turned yellow.” Physical examination is remarkable for jaundice that is most visible in the sclera, palms, and nail beds. Serum chemistry studies show:
- Sodium 141 mEq/L
- Potassium 4.0 mEq/L
- Chloride 102 mEq/L
- Bicarbonate 26 mEq/L
- Urea nitrogen 14 mg/dL
- Creatinine 0.8 mg/dL
- Uric acid 5.1 mg/dL
- Total bilirubin 2.2 mg/dL
- Direct bilirubin 0.3 mg/dL
- Indirect bilirubin 1.9 mg/dL
- Albumin 4.1 g/dL
- Amylase 105 U/L
- AST 20 U/L
- ALT 25 U/L
- Alkaline phosphatase 77 U/L
Which of the following is the most likely diagnosis?
(A) Carcinoma of the ampulla of Vater
(B) Cholesterol gallstone disease
(C) Dubin-Johnson syndrome
(D) Hepatic cirrhosis
(E) Sickle cell disease
Respuesta: E
The correct answer is E. The patient has indirect (unconjugated) hyperbilirubinemia. In adults, the causes include intravascular hemolysis (due to acquired and genetic causes of hemolytic anemia, such as sickle cell anemia, hereditary spherocytosis, glucose-6- phosphate dehydrogenase deficiency, and autoimmune hemolysis) and genetic deficiencies of liver glucuronyl transferase activity (Gilbert syndrome and Crigler-Najjar syndrome). In contrast, direct (conjugated) hyperbilirubinemia can be due to intrahepatic causes (notably hepatitis, drug toxicity, and alcoholic liver disease) or extrahepatic causes (notably common duct stone and pancreatic cancer).
Carcinoma of the ampulla of Vater (choice A) can cause direct hyperbilirubinemia.
Gallstone disease (choice B) involving the common bile duct can cause direct hyperbilirubinemia. Hemolytic anemias predispose for bile gallstones, rather than cholesterol stones.
Dubin-Johnson syndrome (choice C) is a hereditary disease with direct hyperbilirubinemia.
Hepatic cirrhosis (choice D) can cause direct hyperbilirubinemia.
A 45-year-old woman presents to clinic for a health maintenance visit. She has no complaints but has a history of diabetes and a family history of hypertension. On physical examination, the patient’s blood pressure is 150/100 mm Hg. Laboratory results are normal. Which of the following agents would be most appropriate for the management of her hypertension?
(A) Atenolol
(B) Captopril
(C) Furosemide
(D) Hydrochlorothiazide
(E) Isordil
Respuesta: B
The correct answer is B. This question tests your knowledge about the effectiveness of angiotensin converting enzyme (ACE) inhibitors in cardiac remodeling in diabetics. High-dose ACE inhibitors have been shown to have a mortality benefit in hypertensive patients. All the other medicines are good hypertensive alternatives, but captopril is the most effective in diabetics. ACE inhibitors also exert a protective effect on the kidneys in diabetics by inhibiting the actions of angiotensin II on renal afferent arterioles and by attenuating the stimulatory effect of angiotensin II on glomerular cell growth and mesangial matrix production. Of course if the creatinine were high, indicating renal insufficiency, then we would have to re-evaluate our options.
Atenolol (choice A) would be the drug of choice in a patient with coronary artery disease. Beta blockers reduce mortality in such patients by decreasing chronotropy and exerting a protective effect.
Furosemide (choice C) is a loop diuretic used when diuresing a patient in congestive heart failure.
Hydrochlorothiazide (choice D) is a diuretic useful in the management of hypertension, especially in African Americans.
Isordil (choice E) belongs to the nitrate family of vasodilators and exerts its effects as a venodilator that decreases preload. It is used as an adjunct to first-line therapy in hypertension and coronary artery disease.
A 30-year-old man is brought to the emergency department because of acute chest pain for 1 hour. He is admitted with a diagnosis of myocardial infarction, which is confirmed by imaging and serum marker studies. The patient’s family history is significant for early myocardial infarctions in several of his relatives. Nodular lesions are noted in his eyelids and several tendons, which are diagnosed as xanthomas by biopsy examination. Serum cholesterol level is 350 mg/dL, and triglycerides are within the normal range. An abnormality in which of the following proteins most likely accounts for this patient’s condition?
(A) Apolipoprotein CII
(B) Apolipoprotein E
(C) 3-Hydroxy-3-methylglutaryl coenzyme A reductase
(D) LDL receptor
(E) Lipoprotein lipase
Respuesta: D
The correct answer is D. The constellation of high cholesterol level, MI at a young age, positive family history for early-onset MI, and multiple xanthomas is consistent with familial hypercholesterolemia. This autosomal dominant disorder is caused by mutations in the gene for the LDL receptor, resulting in cholesterolemia that is 2-3 times the normal level in heterozygotes and 5-6 times the normal level in homozygotes. The molecular defect leads to impairment in the ability of the liver and other tissues to take up LDL from the plasma. Homozygotes often die in their 20s or 30s because of myocardial or cerebral infarction.
Abnormalities in apolipoprotein CII (choice A) represent the molecular basis for the rare hyperlipoproteinemia type V, characterized by elevation of cholesterol and triglycerides. ApoCII is a major component of VLDL and is removed in the conversion of VLDL into IDL.
Inherited abnormalities of apolipoprotein E (choice B) lead to hyperlipoproteinemia type III, which is characterized by increased IDL and elevated levels of cholesterol and triglycerides. These patients manifest early atherosclerotic disease as well.
3-Hydroxy-3-methylglutaryl coenzyme A reductase (choice C) catalyzes the rate-limiting step in the biosynthesis of cholesterol. There is no known familial abnormality affecting this enzyme.
Lipoprotein lipase (choice E) is present in peripheral tissues (adipose tissue and skeletal muscle) and is involved in the uptake of triglycerides from chylomicrons and VLDL. Abnormalities in this enzyme result in hyperlipoproteinemia types I and IV, characterized by recurrent pancreatitis and hepatosplenomegaly. The atherogenic effects of these abnormalities is minimal, since cholesterol is minimally affected.
A 45-year-old woman comes to the physician because of muscle tenderness, pain on swallowing, hoarseness, and temperatures to 38 C (100 F). Several weeks ago, the patient returned from a trip to Greece, where she ate boar meat. Examination shows muscle swelling and tenderness involving the upper arms, neck, and masseters. Injected conjunctivae and periorbital edema are also noted. Laboratory investigations show:
- Hematocrit 41%
- Leukocyte count 9000/mm3
- Eosinophils 8%
- Erythrocyte sedimentation rate 12/min
- Creatine kinase 2300 U/L
- Albumin 4.0 g/dL
- Globulins 4.3 g/dL
Which of the following is the most likely diagnosis?
(A) Cysticercosis
(B) Dermatomyositis
(C) Polyarteritis nodosa
(D) Systemic lupus erythematosus
(E) Toxoplasmosis
(F) Trichinosis
(G) Trypanosomiasis
Respuesta: F
The correct answer is F. Trichinosis (trichinellosis) is due to the nematode Trichinella spiralis, whose larvae are acquired by ingestion of improperly cooked pork or other types of meat, including boar, bear, and horse. The invasive phase of trichinellosis preferentially affects the skeletal muscle, but also the myocardium, lungs, and brain. A transient intestinal stage (diarrhea and abdominal cramps for a few days) is followed by the muscle invasion stage. The muscles most commonly involved include the masseters, tongue, diaphragm, intercostals, extraocular muscles, laryngeal muscles, nuchal muscles, deltoids, and biceps. Pain and tenderness of involved muscles, periorbital edema, conjunctivitis, hoarseness, and dysphagia are the most characteristic manifestations. Blood eosinophilia, elevated creatine kinase, and a reversed ratio of albumin and globulin are highly characteristic. Serologic tests are available to confirm the clinical diagnosis.
Cysticercosis (choice A) develops following ingestion of Tenia solium eggs in undercooked pork. The brain is one of the preferred sites for cysticercosis, and cysticerci may develop within the brain parenchyma, ventricles, or sub-arachnoid space.
Trichinosis may simulate collagen vascular diseases, such as dermatomyositis (choice B), polyarteritis nodosa (choice C), and systemic lupus erythematosus (choice D). Eosinophilia, however, is consistent with a parasitic disease. Furthermore, collagen vascular diseases are associated with an elevated erythrocyte sedimentation rate, which is usually normal in trichinosis.
Toxoplasmosis (choice E) may affect skeletal muscles, in addition to the CNS, retina, lungs, and myocardium. The cat is the definitive host for Toxoplasma gondii. Primary infection results from ingestion of cysts in inappropriately cooked meat, contaminated vegetables, handling of cat litter, or transplacental infection. The acute infection in immunocompetent hosts is mild and self-limiting. Fever, malaise, myalgia, and cervical lymphadenopathy are characteristic manifestations.
Trypanosomiasis (choice G) causes Chagas disease, endemic in South America. It is transmitted from person to person by triatomids known as “kissing bugs.” Trypanosoma cruzi is an intracellular protozoon that localizes mainly in the heart and nerve cells of the myenteric plexuses, leading to myocarditis, achalasia, megacolon, and megaureter.
A 26-year-old woman with a history of type 1 diabetes presents with nausea and vomiting for 1 day. She has had flulike symptoms for the past 3 days, with mild abdominal pain but no fever or chills. She has cut her insulin dosage in half because she is not eating well. Examination shows her temperature is 37.7 C (99.8 F), blood pressure is 120/80 mm Hg, pulse is 110/min, respirations are 20/min. Examination of the lungs reveals crackles at the right base. Laboratory studies show the following:
- Leukocyte count 13,000/mm3
- Hemoglobin 16.0 g/dL
- Platelets 220,000/mm3
- Sodium 145 mEq/L
- Potassium 4.5 mEq/L
- Chloride 110 mEq/L
- Glucose 322 mg/dL
- Urine Positive for ketones
Determination of arterial blood gases is likely to show which of the following sets of values?
(A) pH 7.33, PCO2 32, HCO3 − 14
(B) pH 7.33, PCO2 40, HCO3 − 15
(C) pH 7.43, PCO2 30, HCO3 − 24
(D) pH 7.45, PCO2 38, HCO3 − 34
(E) pH 7.23, PCO2 47, HCO3 − 18
Respuesta: A
The correct answer is A. This patient has a metabolic acidosis, due to the increase in ketone bodies, with respiratory compensation. (The PCO2 of 32 is below the normal value of 40, indicating hyperventilation in agreement with the increased respiratory rate.)
Choice B represents metabolic acidosis with no respiratory compensation.
Choice C represents respiratory alkalosis, characterized by an isolated decrease in carbon dioxide.
Choice D represents metabolic alkalosis, as evidenced by the increased bicarbonate.
In choice E there is both metabolic acidosis and respiratory acidosis.
A 45-year-old man is seen by a clinician because of chronic gastritis, which he has been self-treating with antacids. Screening blood chemistries demonstrate a plasma phosphate of 2.0 mg/dL. The physician suspects that the antacids may have caused the hypophosphatemia, but the man does not remember which brand of antacid he has been taking. Which of the following common antacid ingredients is most likely responsible for this man’s hypophosphatemia?
(A) Aluminum hydroxide
(B) Calcium carbonate
(C) Magnesium hydroxide
(D) Simethicone
(E) Sodium alginate
Respuesta: A
The correct answer is A Hypophosphatemia is defined as a plasma phosphate less than 2.5 mg/dL. Clinically significant hypophosphatemia occurs in a relatively small number of clinical settings. One of these is a prolonged negative phosphate balance due to binding to aluminum-containing antacids. Other causes include hyperparathyroidism, other hormonal disturbances (Cushing syndrome, hypothyroidism), electrolyte disorders (hypomagnesemia, hypokalemia), theophylline intoxication, chronic diuretic administration, malabsorption, renal dialysis, and starvation.
Calcium carbonate (choice B) commonly causes constipation and can cause increased serum phosphate as part of the milk alkali syndrome.
Magnesium hydroxide (choice C) has a laxative effect and rarely causes hypermagnesemia.
Simethicone (choice D) is sometimes added to antacids to alleviate gas symptoms and is relatively free of side effects.
Sodium alginate (choice E) is sometimes added to antacids and makes a foam that allows smaller antacid doses to be used; it is relatively free of side effects.
A 43-year-old man with alcohol abuse has enrolled in a specialized program for alcohol-related disorders. Which of the following blood tests is useful in monitoring the patient’s compliance with treatment?
(A) Carbohydrate-deficient transferrin
(B) γ-Glutamyl transferase
(C) Mean corpuscular red blood cell volume
(D) Serum triglycerides
(E) Uric acid
Respuesta: A
The correct answer is A. Problem drinking is one of the most common problems coming to the attention of primary care physicians. It encompasses conditions of varying severity, ranging from alcohol dependence (the most serious), to alcohol abuse, and at-risk drinking. History is the key to detection of problem drinking. There are no specific laboratory tests that assist in the diagnosis. However, medical complications related to excessive alcohol consumption can be detected, or suspected, by using certain laboratory parameters. Of the ones listed above, the serum level of carbohydrate-deficient transferrin appears to correlate best with heavy alcohol consumption (more than 5 drinks daily). This is not a useful screening test, but it may be valuable in the follow-up of patients who enrolled in specialized programs for treatment of alcoholism.
Studies show that γ-glutamyl transferase (GGT) levels (choice B) are elevated in more than 30% of problem drinkers, and that providing feedback information about GGT levels to problem drinkers results in a better treatment outcome.
Mean corpuscular red blood cell volume (choice C) is often elevated in chronic alcohol abusers because of frequent coexistence of vitamin deficiency. Vitamin B12 and folic acid deficiencies are the most frequent and cause megaloblastic anemia.
Serum triglycerides (choice D) are also frequently above normal levels (>180 mg/dL) in problem drinkers, who also exhibit high levels of uric acid (choice E) in the blood (>7 mg/dL). Neither test is useful in screening for problem drinking because of low sensitivity.
A 38-year-old woman with rheumatoid arthritis presents to her physician because of increased joint pain. On physical examination, both passive and active range of motion of the hips are decreased. Her physician increases her dose of nonsteroidal anti-inflammatory drugs (NSAIDs). Which of the following should be prescribed as well to prevent peptic ulcer disease?
(A) Cimetidine
(B) Clarithromycin
(C) Misoprostol
(D) Omeprazole
(E) Sucralfate
Respuesta: C
The correct answer is C. NSAIDs inhibit the production of prostaglandins, which are essential for protecting the stomach and duodenum from ulcers. In patients who absolutely require NSAIDs, misoprostol administration will decrease the incidence of peptic ulcer disease (PUD) and upper gastrointestinal bleeding. In patients with a history of these conditions, other agents should be used for pain control if possible.
Cimetidine (choice A) is an H2 blocker that has not proven beneficial for treating NSAID-induced ulcers.
Clarithromycin (choice B) is an antibiotic active against Helicobacter pylori, a known pathogen in PUD. The drug has not been proven beneficial in NSAID-induced ulcers.
Omeprazole (choice D) is a proton-pump blocker. It is not appropriate for treating NSAID-induced ulcers.
Sucralfate (choice E) is a viscous liquid that coats ulcer beds. It has not been proven beneficial in NSAID-induced ulcers.
A 22-year-old college student has had a nonproductive cough, low-grade fever, and severe headache for 6 days, as well as left ear pain for 1 day. His temperature is 38.3 C (101 F), pulse is 78/min, and respirations are 18/min. The left tympanic membrane is erythematous, and there are two small blebs present. Crackles are heard over the right lower lung field. The left lung fields are clear to auscultation. Laboratory studies reveal a white blood cell count of 6000 with a normal differential. A chest xray film reveals a right lower lobe infiltrate with an area of platelike atelectasis. When an anticoagulated tube of blood is cooled with ice, a precipitate forms, clearing with rewarming. Which of the following is the most appropriate treatment?
(A) Amoxicillin
(B) Erythromycin
(C) Gentamycin
(D) Imipenem
(E) Vancomycin
Respuesta: B
The correct answer is B. Erythromycin is the treatment of choice for Mycoplasma pneumonia. Bullous myringitis, which occurs in a small percentage of patients with Mycoplasma infection, is characterized by the presence of erythematous and painful papules on the surface of the tympanic membranes. This patient also has cold agglutinins (classically associated with Mycoplasma), which precipitate on cooling and clear with warming. The IgM antibodies made against Mycoplasma agglutinate RBCs at low temperatures, causing hemolysis in less than 15% of cases. This patient has classic signs and symptoms of Mycoplasma: he is young and has headaches, nonproductive cough, low-grade fever, and infiltrate or atelectasis on chest x-ray
A 40-year-old man presents to a physician because he has been experiencing episodes of severe vertigo accompanied by nausea and vomiting. The first time this happened, he thought he had picked up a gastrointestinal “bug,” but he has now had five of these episodes over the past 6 months. The episodes frequently begin with a sense of fullness in his right ear, which is often accompanied by tinnitus and a sense of hearing loss in the affected ear. Each episode lasts hours to days and then resolves. Otoscopic examination of the affected ear is within normal limits. Which of the following is the most likely diagnosis?
(A) Benign paroxysmal positional vertigo
(B) Herpes zoster oticus
(C) Meniere disease
(D) Purulent labyrinthitis
(E) Vestibular neuronitis
Respuesta: C
The correct answer is C. This is Meniere disease, a disease of poorly understood pathophysiology in which a generalized dilation of the membranous labyrinth of the inner ear (endolymphatic hydrops) is associated with attacks of vertigo, tinnitus, and initially fluctuating and later progressive hearing loss. The clinical description illustrated in the question stem is typical. Meniere disease can affect any age or sex, with a (broad) peak in the 4th and 5th decades of life. Treatment is pharmacologic and often requires some experimentation before medications (e.g., anticholinergics, antihistamines, barbiturates, and diazepam) effective in a particular individual are found.
Benign paroxysmal positional vertigo (choice A) is characterized by violent vertigo induced by moving the head to certain positions.
Herpes zoster oticus (choice B) has prominent pain symptoms in addition to hearing loss, vertigo, and sometimes paralysis of the facial nerve.
Purulent labyrinthitis (choice D) is a bacterial infection of the inner ear and occurs as a complication of acute otitis media or purulent meningitis.
Vestibular neuronitis (choice E) presents with an initial, persistent, severe episode of vertigo that eventually fades to a paroxysmal form, which usually completely disappears within a year or two.
A 37-year-old woman complains of pain during intercourse. Several months ago, she noted the gradual onset of increasing vaginal discomfort, and she is now unable to have intercourse without significant pain. She uses oral contraceptives and does not use condoms or lubricants. During the interview, the patient stops several times to drink from a bottle of water that she carries with her. She reports that she has always had a dry mouth and dry eyes. On physical examination, her temperature is 37.2 C (98.9 F), blood pressure is 110/82 mm Hg, pulse is 74/min, and respirations are 14/min. There is bilateral parotid gland swelling. There are multiple dental caries and fillings. Cardiac examination is significant for a midsystolic click. Which of the following is the most likely diagnosis?
(A) CREST syndrome
(B) Dermatomyositis
(C) Raynaud phenomenon
(D) Scleroderma
(E) Sjögren syndrome
Respuesta: E
The correct answer is E. Sjögren syndrome is an autoimmune disorder resulting in dysfunction of exocrine glands, which leads to dryness of the eyes and mouth, dental caries, dysphagia, and parotid enlargement because of infiltration of these structures with lymphocytes and plasma cells. Dyspareunia may develop, as in this case. There is an association with systemic lupus erythematosus, rheumatoid arthritis, scleroderma, polymyositis, and autoimmune thyroid disease. Treatment is supportive, with artificial tears and dental hygiene. The disease is usually benign, but lymphoma may develop.
CREST syndrome (choice A) differs from scleroderma (systemic sclerosis) in that there is a lower risk of renal involvement, a higher risk of pulmonary hypertension, and an overall better prognosis. CREST patients have thickened skin only on their hands and face, as opposed to the more diffuse skin involvement in scleroderma. Both diseases are characterized by an immunemediated fibrosis of internal organs and skin.
Dermatomyositis (choice B) is a systemic disease of unknown etiology characterized by proximal muscle weakness and a “heliotrope” rash. It is associated with underlying malignancy.
Raynaud phenomenon (choice C) usually affects younger women. The peripheral vasculature exhibits an abnormal response to the cold. Patients experience cyanosis in the fingertips in response to cold or to emotions. Treatment includes keeping the patient’s hands warm with gloves, having the patient refrain from smoking, and, in extreme cases, prescribing calcium channel blockers.
Scleroderma (choice D), or systemic sclerosis, causes hardening and contraction of the connective tissue. The skin is tough and thick and has patches of pigmentation.
A 35-year-old man consults a gastroenterologist because of chronic heartburn for several years. The heartburn tends to be worse at night, and he frequently tastes refluxed gastric contents when he goes to bed. He found that his symptoms were a little better when he avoided his customary late evening alcoholic drink; however, this modest improvement has subsequently deteriorated. The gastroenterologist performs esophageal manometry with pH monitoring, which demonstrates decreased pressure of the lower esophageal sphincter and the presence of acid in the esophagus. Esophagogastroduodenoscopy demonstrates a very irregular gastroesophageal junction with long “fingers” of reddened mucosa extending up to 7 cm above the lower esophageal sphincter. Biopsy of the proximal end of one of these fingers shows surface epithelium with regular columnar cells with small, ovoid nuclei admixed with goblet cells. Which of the following is the most likely diagnosis?
(A) Achalasia
(B) Barrett esophagus
(C) Corrosive esophagitis
(D) Esophageal adenocarcinoma
(E) Zenker diverticulum
Respuesta: B
The correct answer is B. The patient has reflux esophagitis that has become complicated by metaplasia of the squamous mucosa to intestinal-type mucosa (as indicated by the goblet cells). This change, called Barrett esophagus or Barrett mucosa, is considered premalignant since adenocarcinoma of the esophagus can arise in these areas. Recommended treatment of gastroesophageal reflux disease includes elevation of the head of the bed; avoidance of acid stimulators, such as coffee and alcohol; avoidance of agents that decrease lower esophageal sphincter pressure, such as anticholinergics, fats, and chocolate; use of antacids after meals and at bedtime; and use of H2 blockers, prokinetic agents, or protonpump inhibitors, such as omeprazole. Once Barrett esophagus has been demonstrated, endoscopic surveillance to detect developing adenocarcinoma is recommended every 1 or 2 years.
Achalasia (choice A) would show aperistalsis and increased lower esophageal sphincter pressure on manometry.
Corrosive esophagitis (choice C) is seen after accidental or suicidal ingestion of caustic poisons, such as strong cleaning solutions.
Esophageal adenocarcinoma (choice D) is a major complication of Barrett esophagus, but it is not indicated here by the biopsy since the cells seen are regular and nonmalignant.
Zenker diverticulum (choice E) is a posterior outpouching of the esophageal mucosa and submucosa through the cricopharyngeal muscle.
A previously healthy 22-year-old man comes to medical attention because of progressive exertional dyspnea punctuated by episodes of precordial pain. The patient’s father and one of his elder siblings died of chronic heart disease in middle age. The most significant findings on physical examination include presence of a loud S4 and a harsh systolic murmur. The latter increases with the Valsalva maneuver and decreases with squatting. Bibasilar rales are heard on lung auscultation. Blood pressure, pulse, and respiratory rate are within normal limits. A chest x-ray film shows no appreciable alterations in cardiac silhouette, but the ECG shows left axis deviation. Echocardiography reveals marked thickening of the interventricular septum, associated with delayed relaxation and filling of the left ventricle during diastole. Which of the following is the most appropriate initial step in management?
(A) Beta blockers
(B) Digitalis
(C) Long-acting nitrates
(D) Aortic valve replacement
(E) Partial excision of myocardial septum
Respuesta: A
The correct answer is A. The clinical presentation is consistent with hypertrophic cardiomyopathy, a frequently inherited disorder due to mutations in one of the genes encoding myofibrillary proteins. Left ventricular dysfunction is due to impaired diastolic filling. The septum is disproportionately thick compared with the free wall (asymmetric hypertrophy), resulting in outflow obstruction. The latter is the cause of the systolic murmur, which intensifies with decreased left ventricular filling, such as during the Valsalva maneuver. Beta blockers or calcium channel blockers should be the initial treatment.
Digitalis (choice B) is contraindicated in hypertrophic cardiomyopathy because it enhances myocardial contractility and, consequently, the degree of outflow obstruction secondary to increased septal contraction.
Long-acting nitrates (choice C) are used for treatment of angina and related coronary artery syndromes, but not for chest pain secondary to hypertrophic cardiomyopathy. The latter, in fact, is not due to vascular stenosis but to a combination of decreased left ventricular output and increased oxygen demands of the hypertrophic myocardium.
Aortic valve replacement (choice D) is indicated for treatment of aortic stenosis, but the aortic valve is normal in this condition. Outflow obstruction is the result of a thick septum that partially obstructs the outflow tract.
Partial excision of the myocardial septum (choice E) has been successful in some centers but should be reserved for severe cases in which pharmacologic therapy has failed.
A 47-year-old alcoholic man is picked up by the police and taken to the emergency department. The police, who have had previous encounters with this man, are concerned that he seems particularly lethargic and confused. Stat blood chemistries demonstrate a plasma sodium of 115 mEq/L. The emergency department physician orders the administration of hypertonic saline; within 4 hours, the plasma sodium rises to 135 mEq/L. Over the next few days, the patient develops quadriparesis and weakness of the lower face and tongue. These symptoms never resolve. Damage to which of the following neural structures probably accounts for these findings?
(A) Cerebellum
(B) Cerebral cortex
(C) Peripheral nerves
(D) Pons
(E) Spinal cord
Respuesta: D
The correct answer is D. This is central pontine myelinolysis, which is a feared complication of severe hyponatremia. This demyelinating condition of the pons often produces permanent damage, which may manifest as illustrated in the question stem. More severe cases can even cause a locked-in syndrome, in which the patient is in an awake and sentient state but has complete, generalized motor paralysis, leaving only perhaps a limited ability to communicate by coded eye movements. (Fiction aficionados may recall that the character Noirtier in The Count of Monte Cristo had this syndrome. He could communicate only with his granddaughter, who was the only person patient enough to talk to him and interpret his eye blinks.) Controversial evidence suggests that overly rapid correction of severe hyponatremia increases the chances that the complication will develop, possibly because of fluid shifts in the confined area of the pons. It is now recommended that plasma sodium be raised no faster that 1 mEq/L/hr, with an upper limit of 10 mEq/L/24 h.
Although demyelination can also involve other areas of the brain to much lesser degrees, the cerebellum (choice A), cerebral cortex (choice B), peripheral nerves (choice C), and spinal cord (choice E) are not usually as significantly damaged as the pons in this condition.
A 61-year-old woman presents to her physician’s office for a routine physical. She has a history of diabetes and hypertension and has a 30-pack-year history of cigarette smoking. Laboratory studies show:
- Sodium 136 mEq/L
- Potassium 4.5 mEq/L
- Chloride 108 mEq/L
- BUN 24 mg/dL
- Creatinine 0.9 mg/dL
- Calcium 11.5 mg/dL
- Albumin 3.4 g/dL
Which of the following is the most likely diagnosis?
(A) Excess vitamin D intake
(B) Occult malignancy
(C) Paget disease
(D) Parathyroid hormone (PTH) oversecretion
(E) Sarcoidosis
Respuesta: D
The correct answer is D. The most common cause of asymptomatic hypercalcemia is primary hyperparathyroidism. In older women, 85% of cases are due to adenoma of a single gland, 15% are caused by hyperplasia of all four parathyroid glands, and about 1% are associated with carcinoma. Most patients have asymptomatic hypercalcemia that is found incidentally during a routine laboratory examination.
Excess vitamin D intake (choice A) is an uncommon cause of hypercalcemia.
Occult malignancies (choice B) are responsible for most cases of “in hospital” hypercalcemia. Underlying mechanisms include release of a PTH-like substance and local osteolytic hypercalcemia.
Paget disease (choice C) is associated with an increase in alkaline phosphatase and normal calcium and phosphate.
Sarcoidosis (choice E) is associated with hypercalcemia due to increased production of 1,25-(OH)2 vitamin D3 by alveolar macrophages, but it is less common. This phenomenon also occurs in other chronic granulomatous disorders, in lymphomas, and in idiopathic hypercalciuria
A 35-year-old man with recurrent, active, genital herpes suddenly develops numerous erythematous macules, papules, wheals, and vesicles. The hands, feet, and face are most extensively involved. Careful examination demonstrates that many of the lesions have a “target” appearance. Some of the lesions involve the lips and buccal mucosa. Which of the following is the most likely diagnosis?
(A) Erythema multiforme
(B) Erythema nodosum
(C) Granuloma annulare
(D) Pemphigus
(E) Toxic epidermal necrolysis
Respuesta: A
The correct answer is A. The patient has erythema multiforme, which is an inflammatory eruption characterized by lesions showing a variety of morphologies that may involve both skin and mucous membranes. Erythema multiforme can occur idiopathically; as a complication of viral infections (notably herpes simplex viruses, but also coxsackievirus and echovirus), Mycoplasma pneumoniae infection, or fungal infections (histoplasmosis); or as reaction to a drug (penicillin, sulfonamides, barbiturates) or vaccine (BCG, vaccinia, poliomyelitis). Erythema multiforme tends to have a sudden onset, with a predilection for most severe involvement of the hands (notably palms), feet (notably soles), and face (notably near or involving mucous membranes). The most helpful lesion morphology is the target lesion, with an erythematous ring, midpallor, and central erythematous macule; it may, in practice, be necessary to look at a number of lesions before a target lesion is identified. Attacks of erythema multiforme tend to last 2-4 weeks; treatment is usually primarily directed against any underlying condition or drug that triggered the skin reaction.
Erythema nodosum (choice B) usually causes tender red nodules of the pretibial areas of the legs.
Granuloma annulare (choice C) causes a peripheral ring of nodules around normal to slightly depressed skin.
Pemphigus (choice D) causes flaccid blisters of skin and mucous membranes.
Toxic epidermal necrolysis (choice E) causes sheets of skin to peel off, analogous to the peeling skin of sunburn.
A 60-year-old woman comes to the physician because of jaundice, pruritus, and anorexia for 2 weeks. She has one or two alcoholic drinks on social occasions and has smoked one pack of cigarettes daily for 30 years. She is currently taking a thiazide diuretic for mild hypertension. Her temperature is 36.8 C (98 F), blood pressure is 130/80 mm Hg, pulse is 80/min, and respirations are 14/min. Physical examination confirms icteric discoloration of skin and mucosae. Abdominal examination is remarkable for slight tenderness in the right upper quadrant, and the liver is palpable 1 cm below the right costal arch. The spleen is not palpable. Serum chemistry tests show:
- AST 60 U/L
- ALT 40 U/L
- Alkaline phosphatase 1000 U/L
- Total bilirubin 5.5 mg/dL
- Direct bilirubin 4.0 mg/dL
Which of the following is the most appropriate next step in diagnosis?
(A) Abdominal ultrasound
(B) Abdominal CT or MRI scan
(C) Endoscopic retrograde cholangiopancreatography
(D) Percutaneous liver biopsy
(E) Percutaneous transhepatic cholangiography
Respuesta: A
The correct answer is A. The most crucial step in beginning investigations on a jaundiced patient is to determine whether the jaundice is due to hemolytic disease, hepatocellular damage, or biliary obstruction. Generally, jaundice resulting from hemolysis or hepatocellular damage must be treated with medical measures, whereas obstructive jaundice requires surgical treatment. Obstructive jaundice is characterized by a high proportion of direct (conjugated) bilirubin, elevated serum alkaline phosphatase levels, and normal or mildly elevated aminotransferases. This case is therefore due to biliary obstruction. How to proceed? The most appropriate investigation is ultrasound examination to evaluate the hepatobiliary system and pancreas, determine whether extrahepatic bile ducts are dilated, establish the presence of gallstones, and identify hepatic or pancreatic masses.
CT or MRI scans (choice B) are also adequate but are more expensive; thus, ultrasound constitutes the most convenient initial diagnostic test.
Endoscopic retrograde cholangiopancreatography (ERCP; choice C) is the method of choice to study pancreatic and ampullary lesions, carry out stone extraction from the common bile duct, or insert a stent.
Percutaneous liver biopsy (choice D) is the definitive study for hepatocellular or infiltrative diseases. It is not used as an initial diagnostic procedure because of its invasiveness and potential adverse effects.
Percutaneous transhepatic cholangiography (choice E) can identify the location of biliary obstruction, but may be associated with serious complications, i.e., bacteremia, bile peritonitis, and hemorrhage. At any rate, this procedure should follow other noninvasive radiologic methods.
A 45-year-old woman presents with recent onset of low back pain for the past 3 days. She has not sustained any significant trauma. She has a history of systemic lupus erythematosus and has been receiving chronic corticosteroid treatment. She is currently afebrile. Examination reveals tenderness on palpation of the lumbar spine. The pain does not radiate down the leg. Which of the following is the most likely diagnosis?
(A) Ankylosing spondylitis
(B) Cauda equina syndrome
(C) Compression fracture
(D) Herniated intervertebral disk
(E) Infection
(F) Neoplasm
(G) Spinal stenosis
Respuesta: C
The correct answer is C. From a primary care perspective, the great majority of cases of low back pain are due to degenerative joint disease. However, the primary care physician should be alert to “red flags” that may signal the presence of more serious pathology. A history of corticosteroid treatment may be associated with osteoporosis of the vertebral column, which predisposes patients to compression fractures of the vertebral bodies. This type of fracture may present with back pain without any apparent history of preceding trauma.
Ankylosing spondylitis (choice A) is characterized by onset before 40 years of age, progressive ankylosis of the vertebral column, and bilateral involvement of the sacroiliac joints. More than 90% of cases are associated with HLA-B27 histocompatibility antigen. This disease should be suspected in a young person complaining of chronic lower back pain, especially when the symptoms are worst at night.
Cauda equina syndrome (choice B) is an acute clinical picture of lumbar root compression. It is a surgical emergency due to acute herniation of an intervertebral disk or other anatomic abnormalities and is characterized by the sudden onset of pain, saddle anesthesia, and bowel/bladder incontinence.
A herniated intervertebral disk (choice D) manifests with the acute onset of low back pain, which radiates down the leg in the distribution of a spinal root. The most common sites of disk herniation are the intervertebral spaces between L4-L5 and L5-S1. Sensory and motor testing usually reveals neurologic deficits in a dermatomal distribution.
Infection (choice E) of the vertebral bodies or joints (spinal osteomyelitis) is most prevalent among IV drug abusers. Fever and constitutional symptoms are associated with back pain.
A neoplasm (choice F) should be suspected if there is concomitant weight loss or other constitutional symptoms (e.g., anorexia, malaise, or fatigue). Metastases and multiple myeloma are the most common malignancies affecting the spinal column.
Spinal stenosis (choice G), an infrequent cause of low back pain, is more prevalent in the elderly population. Narrowing of the lumbar spinal canal by osteophytes is thought to account for the symptomatology. Pain due to compression of lumbar roots manifests while in the upright position or walking (pseudoclaudication), when the spinal canal becomes narrower. The pain is relieved by sitting or leaning forward, as the spinal canal widens.
A 40-year-old man with a history of IV drug use presents with a 2- week history of high-grade fevers. He has been seen in clinic before and is now found to have a new murmur on cardiac examination. He also has several dark painful spots on his feet. Which of the following is the most common cause of negative blood cultures in patients with this illness?
(A) Fungal infection
(B) Inadequate culture techniques
(C) Prior administration of antibiotics
(D) Prosthetic valve seeding
(E) Right-sided endocarditis
Respuesta: C
The correct answer is C. This patient most likely has infective bacterial endocarditis. Ill-advised use of antibiotics, before samples for blood culture are obtained, is a serious problem in the diagnosis and treatment of infections and is a leading cause of the increase in drug-resistant pathogens.
Fungal infections (choice A) can cause endocarditis in IV drug abusers, diabetics, and immunocompromised hosts, but bacterial infections are more frequent in IV drug abusers.
Blood cultures usually yield results (compare with choice B) if the samples are obtained under sterile conditions. With positive blood cultures, however, it is important to ensure that the cultures are not falsely positive because of contamination.
Prosthetic valve seeding (choice D) is relatively common. In fact prosthetic valves, bicuspid aortic valves, and mitral valve prolapse are risk factors for bacterial endocarditis, rather than the cause of negative cultures.
Right-sided endocarditis (choice E) should still yield positive blood cultures. Right-sided infection can lead to emboli to the lungs, causing hemoptysis, and right-sided cardiac failure.
A 45-year-old woman is taken to the emergency department with severe, colicky right upper quadrant pain for the past 2 days. She is on medications and denies alcohol use. Ultrasound studies show gallstones, and the patient is taken for cholecystectomy the next morning. Intraoperatively, it is noted that the liver has a yellowish color, and a liver wedge biopsy is submitted along with the gallbladder to pathology. The pathologist reports steatosis with a predominately macrovesicular pattern in the liver. Which of the following is the most likely explanation for these findings?
(A) Breast cancer
(B) Chronic pancreatitis
(C) Diabetes mellitus
(D) Peptic ulcer disease
(E) Systemic lupus erythematosus
Respuesta: C
The correct answer is C. Although steatosis (fatty liver) has a wide variety of causes, the three most commonly encountered in clinical medicine in the U.S. are alcohol use, obesity, and diabetes mellitus. Rarer causes include jejunal bypass surgery, malnutrition, and as a complication of some drugs (e.g., glucocorticoids, synthetic estrogens, amiodarone, tamoxifen). The specific etiology of the steatosis cannot be determined reliably by pathologic examination, so clinical correlation is necessary. Cases may be picked up incidentally, as in this case, or during liver biopsy performed in patients for other reasons, e.g., in asymptomatic patients with a twoto threefold increase in AST or ALT.
Tamoxifen therapy of breast cancer (choice A) is an unusual cause of fatty liver.
Chronic pancreatitis (choice B) does not cause fatty liver, although both can coexist in alcoholics.
Peptic ulcer disease (choice D) does not cause fatty liver.
Glucocorticoid therapy in systemic lupus erythematosus (choice E) is an unusual cause of fatty liver
A 52-year-old woman consults a physician because of severe epigastric pain of several months’ duration. Endoscopy demonstrates thickened gastric mucosa with several peptic ulcers in the stomach and duodenum. The basal acid secretion rate is 75% of that following a maximal stimulating dose of histamine. Serum gastrin levels are markedly elevated (1200 pg/mL). Endoscopic ultrasound demonstrates a pancreatic mass, which is later resected. Following the resection, serum gastrin levels decrease to 50 pg/mL. In addition to the tumor that was resected, this patient would most likely develop which of the following during her lifetime?
(A) Marfanoid habitus
(B) Medullary carcinoma of the thyroid
(C) Mucosal neuromas
(D) Pheochromocytoma
(E) Pituitary adenoma
Respuesta: E
The correct answer is E. The first step in solving this question is to recognize that the patient has severe PUD in the setting of markedly increased basal acid secretion by the stomach and markedly elevated levels of serum gastrin. This suggests ZollingerEllison syndrome, in which an endocrine tumor secretes gastrin. All Zollinger-Ellison patients have serum gastrin levels greater than 150 pg/mL. Markedly elevated levels (>1000 pg/mL) in the setting of compatible clinical features and gastric acid hypersecretion (>60% of the amount of acid after a maximal stimulating dose of histamine) is considered diagnostic. In this case, the hypergastrinemia comes from a pancreatic endocrine tumor; other sites can include the duodenum, splenic hilum, mesentery, stomach, or even lymph nodes or ovary. More than 50% of gastrinomas are malignant. About half the patients with Zollinger-Ellison syndrome have multiple tumors, usually as part of multiple endocrine neoplasia type I (MEN I, or Wermer syndrome). In MEN I, pancreatic islet cell tumors are found in 30 to 75% of patients; parathyroid adenomas in more than 90%; and pituitary adenomas in 50 to 65%. MEN I patients may also rarely have duodenal gastrinomas.
MEN IIA, or Sipple syndrome, is characterized by medullary carcinoma of the thyroid (choice B; more than 90% of affected patients), pheochromocytomas (choice D; 50%), and parathyroid adenomas (25%).
MEN IIB, or mucosal neuroma syndrome, is characterized by mucosal neuromas (choice C; close to 100% of affected patients), Marfanoid habitus (choice A; close to 100%), medullary carcinoma of the thyroid (choice B; more than 90%), and rarely parathyroid adenomas.
A confused elderly man is taken to the emergency department. The patient was found wandering and complaining of severe abdominal pain, but is unable to give a coherent history. Urine dipstick demonstrates positive ketones but no glucosuria. Stat blood chemistries show a plasma glucose of 100 mg/dL. Which of the following is the most likely explanation for these findings?
(A) Alcoholism
(B) Congestive heart failure
(C) Emphysema
(D) Inflammatory bowel disease
(E) Rheumatoid arthritis
Respuesta: A
The correct answer is A. Be aware that ketoacidosis can be seen in conditions other than diabetes mellitus. The two most important are starvation and chronic alcoholism. The mechanism of ketone production in chronic alcoholics appears to be a combination of alcohol withdrawal and starvation with increased free fatty acid release. The typical history (when it can be obtained) is that of an alcoholic binge that ended with vomiting, after which the patient consumed neither food nor water for 24 hours or more. Abdominal pain is usually a prominent complaint. Many of these patients have pancreatitis, and impaired glucose tolerance or even mild type 2 diabetes mellitus are often demonstrated after the patients recover from their acute episode.
Congestive heart failure (choice B), emphysema (choice C), and rheumatoid arthritis (choice E) do not cause nondiabetic ketoacidosis.
Although severe inflammatory bowel disease (choice D) could in theory cause enough “starvation” to induce ketoacidosis, in practice this is not usually seen.
A 70-year-old woman with metastatic lung cancer is brought to the hospital for increasing confusion and obtundation over the past several weeks. Her family denies recent head trauma. The woman is currently receiving chemotherapy for her malignancy, but no other medications. On physical examination, her vital signs are stable, her jugular venous pressure (JVP) is 7 cm H2O, and her lungs are clear. Her heart rhythm is regular, and she has no edema in her legs. Laboratory analysis shows:
- Sodium 124 mEq/L
- Potassium 4.5 mEq/L
- Chloride 109 mEq/L
- Bicarbonate 25 mEq/L
A CT scan of her head indicates no brain metastases. What is the most appropriate next step in management?
(A) Administer diuretics and water
(B) Administer isotonic saline
(C) Administer water
(D) Restrict intake of salt
(E) Restrict intake of water
Respuesta: E
The correct answer is E. This patient has euvolemic hyponatremia, which means that her effective circulating volume is normal. Her euvolemic status is supported by her normal jugular venous pressure (JVP) of 7 cm water, an indication of her intravascular status. Patients in congestive heart failure would have an elevated JVP. Furthermore, her lungs are clear, and she has no leg edema. Euvolemic hyponatremia can be caused by a glucocorticoid defect, hypothyroidism, or inappropriate secretion of arginine vasopressin (antidiuretic hormone; ADH). SIADH can be caused by many things, including trauma, infections, medications, and certain neoplasms (ectopic production), including lung cancer. Excess ADH secretion leads to retention of free water, producing hyponatremia. The first step in managing this would be to restrict the intake of water, producing a negative water balance that results in gradual, daily reduction in weight and a progressive rise in serum sodium.
Giving diuretics and water (choice A) is a therapeutic option for treating hypernatremia. The diuretic would lead to the removal of excess salt, which would not be desirable.
Administering isotonic saline (choice B) is the equivalent of giving more free water, which will be retained, and the salt will be excreted, thus worsening the hyponatremia.
Administering water (choice C) will worsen the hyponatremia since the patient is simply being given free water. This treatment would be appropriate in treating hypernatremia.
This patient is hyponatremic since her effective salt load is low. Restricting the salt intake would thus not help (choice D). In fact, allowing her to take salt is also not likely to be effective in controlling the hyponatremia since ADH leads to excess retention of free water at the cost of sodium excretion.
An 18-year-old female athlete undergoes successful arthroscopic repair of a torn right medial meniscus. She has been ambulating with the aid of crutches and now complains of profound weakness of her right arm. Physical examination is normal except for an inability to actively extend the right arm at the elbow or the right hand at the wrist. The muscle tone, reflexes, flexors, and intrinsic hand muscles are normal. There is a minor sensory deficit over the dorsolateral area of the right hand. Which of the following is the most likely diagnosis?
(A) Axillary nerve palsy
(B) Dorsal scapular nerve palsy
(C) Median nerve palsy
(D) Radial nerve palsy
(E) Ulnar nerve palsy
Respuesta: D
The correct answer is D. The radial nerve originates from C5 -T1 and innervates the triceps, brachioradialis, wrists, and finger and thumb extensors. The lesion usually affects the spiral groove of the humerus. Clinical features include wrist drop and inability to extend the elbow and fingers because of severe denervation of the dorsolateral hand. Radial nerve palsy is sometimes known as “Saturday night palsy,” since it is found in patients who have been drinking and fall asleep with an arm hanging over a chair. Patients on crutches may also pinch the nerve with their crutches, producing a similar palsy.
The axillary nerve, C5 -C6 (choice A), innervates the deltoid muscle and teres minor. The symptoms of axillary nerve palsy include weakness in shoulder abduction. The injury usually occurs near the shoulder joint.
Dorsal scapular nerve (C5 ) palsy (choice B) leads to dysfunction of the levator scapulae and rhomboid muscles.
Palsy of the median nerve, C6 -T1 (choice C), which innervates the abductor pollicis brevis, forearm, pronator, and finger and thumb flexors, may be due to carpal tunnel syndrome (entrapment of the median nerve in the flexor retinaculum at the wrist). Patients classically are unable to make a circle with their thumb and index finger because of sensory paresthesias in the ventral aspect of the lateral 2 1/2 fingers. It is commonly seen in workers who spend hours at a time at a keyboard.
The ulnar nerve, C8 -T1 (choice E), innervates the flexors of the wrist, the flexors of the fourth and fifth digits, and most intrinsic hand muscles. With ulnar nerve palsy, the injury usually occurs at the ulnar groove in the elbow. The patient may display “claw hand,” with weakness of finger adduction and abduction, and thumb adduction and abduction.
A 45-year-old woman presents with a yellowish discoloration of her body, first noted by her husband last week. Since then, she has been having severe itching at night, which disturbs her sleep, and complains of a tingling sensation of her hands and feet. On examination, xanthelasmas are seen around the eyes. The liver is firmly palpable 4 cm below the costal margin. Scratch marks are noted on her abdomen and limbs. Clubbing is observed in all the digits. Serum creatinine is 0.9 mg/dL, bilirubin is 2.3 mg/dL, albumin is 4.3 g/dL, alanine aminotransferase is 92 U/L, and alkaline phosphatase is 410 U/L. Which of the following is the most appropriate next step in diagnosis?
(A) Anti-mitochondrial antibody assay
(B) Anti-smooth muscle antibody assay
(C) Endoscopic retrograde cholangiopancreatography (ERCP)
(D) Serum protein electrophoresis
(E) Technetium (99mTc) liver-spleen scan
Respuesta: A
The correct answer is A. The patient probably has primary biliary cirrhosis. This disorder of unknown etiology appears to involve an autoimmune response to mitochondrial antigens. Autoantibodies in these patients have been found that recognize mitochondrial inner membrane proteins, such as enzymes of the pyruvate dehydrogenase complex, the branched chain alphaketoacid dehydrogenase complex, and the alpha-ketoglutarate dehydrogenase complex. The antimitochondrial antibody assay is the best test to diagnosis primary biliary cirrhosis. An IgG autoantibody directed against mitochondrial antigens is present in 90% of affected patients.
Anti-smooth muscle antibody (choice B) is positive in some patients with autoimmune chronic hepatitis, but is not specific or highly useful in the diagnosis of primary biliary cirrhosis.
ERCP (choice C) is better used in the diagnosis and monitoring of secondary biliary cirrhosis due to prolonged biliary tract obstruction.
Serum protein electrophoresis (choice D) might show a diffuse increase in immunoglobulins, but this is nonspecific and may be found in many chronic liver diseases.
The sulfur colloid technetium ( 99mTc) scan (choice E) is useful for detecting portal hypertension and hypersplenism, but it is not specific.
A 41-year-old man with alcoholic cirrhosis is admitted to the hospital for evaluation of increasing abdominal girth. The patient has a 3-year history of cirrhosis due to 20 years of ethanol consumption. He has no known cardiac disease but has had two upper gastrointestinal bleeds secondary to portal hypertension from his cirrhosis. He underwent variceal banding on both occasions. He reports that since his last admission for ascites 5 months ago, he has continued to drink ethanol but has been taking his daily spironolactone and furosemide. On physical examination, his abdomen is distended with occasional spider angiomata and a fluid wave on palpation. A diagnostic paracentesis is planned. Monitoring which of the following laboratory values is most important in the continued care of this patient?
(A) Bleeding time
(B) Hematocrit
(C) Partial thromboplastin time (PTT)
(D) Platelet count
(E) Prothrombin time (PT)
Respuesta: E
The correct answer is E. With any invasive procedure, there are risks. One such risk is that of bleeding. Patients with liver disease have varying degrees of damage to their liver parenchyma and thus less synthetic ability. Liver patients often exhibit coagulopathies for this reason; their prothrombin time (PT) can exceed 19 seconds. Although the risk of bleeding is not directly related to a prolonged PT, it is positively associated, so this number is crucial to know prior to performing any procedure on a patient. A number of studies indicate that paracentesis may be safely performed in patients with a PT international normalized ratio (INR) up to 5.0.
Bleeding time (choice A) is a marker of platelet function or number. This is not a laboratory value frequently obtained today since its utility is unclear.
Hematocrit (choice B) is not a necessary value to know prior to performing this procedure. It will not effect current management whether the value is increased or decreased.
Although the PTT (choice C) is also mildly prolonged in liver disease, this is not a risk factor for bleeding episodes associated with invasive procedures.
It is a generally accepted practice to order a platelet count (choice D) prior to a paracentesis. This is not the most useful hematologic value to obtain; however, profound thrombocytopenia (<10,000/mm3 ) can be associated with an increased risk of bleeding after invasive diagnostic procedures.
A 53-year-old man collapses while in the checkout line at the supermarket. A bystander administers CPR until an ambulance arrives after approximately 10 minutes. During the ride to the hospital, the pulse is lost twice, and further resuscitation and electric defibrillation are required. At the emergency department, he has a steady sinus rhythm and normal, stable, vital signs, but he is in deep coma, with bilateral fixed, dilated pupils. His past medical and surgical history are unknown. Which of the following might ultimately improve his neurologic outcome?
(A) Anticoagulation with heparin drip
(B) High-dose systemic steroids (dexamethasone)
(C) Hyperventilation to a PCO2 of 25 mm Hg
(D) Moderate hypothermia to 33.0 C (91.4 F) core temperature
(E) Sedation with barbiturates
Respuesta: D
The correct answer is D. There is actually no magic treatment to undo the neurologic damage done by prolonged cardiac arrest, but there is some evidence that moderate hypothermia improves the chances for a better outcome.
Anticoagulation (choice A) has nothing to offer in this setting.
Steroids (choice B) are used often in neurologic situations. There is some evidence that they may help patients with spinal cord injury, and there is well-documented dramatic benefit in lowering intracranial pressure in patients with brain tumors, but in this particular setting there is no indication for using them.
Hyperventilation (choice C) has been used in the past in a prophylactic way, in situations in which brain edema is expected. The current consensus is that it should not be used until increased intracranial pressure has been diagnosed.
Sedation (choice E) has been used as a last-resort measure in cases of increased intracranial pressure, with the idea of decreasing the metabolic needs of the brain at a time when the oxygen supply might be compromised. Currently, hypothermia is preferred to achieve the same objective.
A 28-year-old man comes to medical attention because of sustained hypertension for the past year. During the same period, he also had paroxysmal episodes characterized by profuse sweating, headache, and a sensation of increased heart rate. At this time, his blood pressure is 160/95 mm Hg, with a pulse of 86/min on supine position. On standing, his blood pressure is 120/70 mm Hg and pulse is 110/min. A dipstick test reveals glucose in the urine. Which of the following is the most appropriate next step in diagnosis?
(A) CT and/or MRI studies of the abdomen
(B) Measurement of epinephrine and norepinephrine in the blood
(C) Measurement of fasting glucose levels in plasma or serum
(D) Pharmacologic provocative tests
(E) Urinary assay for free catecholamines and vanillylmandelic acid
Respuesta: E
The correct answer is E. The history of hypertension punctuated by paroxysmal episodes of hypertensive crises is highly suggestive of pheochromocytoma, a tumor of the adrenal medulla or paraganglia that secretes large amounts of epinephrine and/or norepinephrine, resulting in hypertension. These hormones also cause hyperglycemia and, consequently, glycosuria. Note the orthostatic hypotension and tachycardia, which are frequent manifestations of pheochromocytoma. The most appropriate initial diagnostic test is to measure catecholamines and vanillylmandelic acid (a metabolic product of catecholamines) in the urine. An overnight urine collection is generally sufficient for this purpose.
CT and/or MRI studies of the abdomen (choice A) should be performed for localization of pheochromocytoma once the diagnosis has been supported by the previous test.
Measurement of epinephrine and norepinephrine in the blood (choice B) is the most sensitive test, but needs to be carried out during paroxysmal episodes of hypertension.
Measurement of fasting glucose levels in plasma or serum (choice C) represents an ancillary test that is not indispensable for a diagnosis of pheochromocytoma. It is certainly mandatory in suspected cases of diabetes mellitus.
Pharmacologic provocative tests (choice D) are no longer recommended. These are based on drugs that provoke release of catecholamines, raising blood pressure, or block adrenergic receptors, lowering blood pressure.
A 27-year-old man is brought to the hospital with a gunshot wound of the abdomen, and he is prepared quickly for emergency laparotomy. The endotracheal intubation is achieved with the help of succinylcholine, and he is then switched to nondepolarizing agents and inhaled halothane. Shortly thereafter, the anesthesiologist notices tachycardia, hypertension, and increased CO2 production; subsequently the core body temperature rises to 40.6 C (105.0 F) and hyperkalemia develops. Which of the following is the intrinsic pathophysiology responsible for these abnormalities?
(A) Bacteremia
(B) Massive release of catecholamines
(C) Massive release of thyroid hormone
(D) Sustained muscular contracture
(E) Unrecognized adrenal insufficiency
Respuesta: D
The correct answer is D. The clinical picture is that of malignant hyperthermia that is often triggered in susceptible patients by succinylcholine. The intrinsic problem is sustained muscular contracture.
Bacteremia (choice A) could account for rapid temperature rise, but would not produce hypertension and hyperkalemia.
Catecholamine release (choice B) in a patient not previously diagnosed with pheochromocytoma is most impressive for the amounts of arterial hypertension that rapidly develop. Massive temperature elevation is not a prominent feature.
A thyroid storm (choice C) can produce tachycardia and hyperthermia, but the setting would typically be that of a known patient with hyperthyroidism who is subjected to stress.
Adrenal insufficiency (choice E) would produce hyperkalemia, but the other manifestations include hyponatremia and hypotension. Temperature elevation is not a feature.
A 25-year-old woman presents with a 2-month history of recurrent transient episodes of visual blurring, dizziness, and focal neurologic deficits, such as limb weakness and paresthesias. She also reports numbness and coldness of her fingers. Physical examination discloses marked weakening of the pulses in the upper extremities. An aortic angiogram reveals proximal narrowing of the brachiocephalic, left common carotid, and left subclavian arteries. The aortic arch appears normal.
(A) Aortic dissection
(B) Atherosclerotic disease
(C) Churg-Strauss syndrome
(D) Cryoglobulinemic vasculitis
(E) Giant cell arteritis
(F) Henoch-Schönlein purpura
(G) Kawasaki syndrome
(H) Microscopic polyangiitis
(I) Polyarteritis nodosa
(J) Syphilitic aortitis
(K) Takayasu arteritis
(L) Thromboangiitis obliterans
(M) Wegener granulomatosis
Respuesta: K
The correct answer is K. Takayasu arteritis, also known as pulseless disease because of the frequent absence of pulses in the upper extremities, was first described in Asian countries but has been reported in the U.S. as well. This inflammatory arteritis affects the main branches originating from the aortic arch, causing signs and symptoms of ischemia to the brain and upper extremities. Young women are most commonly affected
Aortic dissection (choice A) manifests with chest pain of sudden onset radiating to the back, signs of shock, and severe distress.
Atherosclerotic disease (choice B) would not explain any of these clinical presentations.
Churg-Strauss syndrome (choice C) is a vasculitis affecting small vessels of lungs, kidneys, skin, and other organs. It is usually associated with eosinophilia and p-ANCA antibodies.
Cryoglobulinemic vasculitis (choice D) results from deposition of immune complexes composed of immunoglobulins that precipitate at cold temperatures in capillaries and venules. The skin is frequently involved.
Giant cell arteritis (choice E) most commonly affects the branches of the external carotid artery, especially the temporal artery. It manifests in middle-aged or elderly persons with headache, fever, and malaise. It may lead to blindness if not promptly treated.
Henoch-Schönlein purpura (choice F) is associated with deposition of immune complexes containing IgA in the skin (purpura), gastrointestinal tract (abdominal pain and melena), joints (arthralgia), and glomeruli (hematuria).
Kawasaki syndrome (choice G) predominantly affects children. Cervical lymphadenopathy, inflammation of conjunctiva and oral mucous membranes, a desquamative rash typically involving the palms, and fever are the most typical manifestations. Vasculitis of coronary arteries occurs in 80% of cases and is the most common cause of death.
Microscopic polyangiitis (choice H) is a vasculitis similar to polyarteritis nodosa but results from an immune response to circulating agents, such as drugs (antibiotics) or infectious agents. Circulating p-ANCA are frequently found.
Polyarteritis nodosa (choice I) may affect medium-sized arteries of kidneys, heart, gastrointestinal tract, joints, muscles, and central and peripheral nervous systems. It is not associated with circulating ANCA.
Syphilitic aortitis (choice J) is a complication of the tertiary stage of syphilis. It results from endarteritis obliterans involving the vasa vasorum of the ascending aorta, which undergoes scarring and dilatation with consequent aortic regurgitation.
Thromboangiitis obliterans (choice L) occurs almost exclusively in young male smokers. It is an inflammatory condition affecting the neurovascular bundles in the limbs and causing ischemia, venous thrombosis, and pain due to nerve involvement.
A 34-year-old woman presents with a 6-month history of sinusitis and otitis media unresponsive to common antibiotic and antiinflammatory treatments. She recently developed increasing malaise and a persistent cough productive of blood-tinged sputum. Her temperature is 38.2 C (100.8 F), blood pressure is 126/81 mm Hg, pulse is 80/min, and respirations are 16/min. Laboratory studies show mild anemia and leukocytosis, an erythrocyte sedimentation rate (ESR) of 87/min, and the presence of circulating antineutrophil cytoplasmic antibodies of c-ANCA type. Urinalysis shows more than 5 red blood cells/high-power field and red cell casts.
(A) Aortic dissection
(B) Atherosclerotic disease
(C) Churg-Strauss syndrome
(D) Cryoglobulinemic vasculitis
(E) Giant cell arteritis
(F) Henoch-Schönlein purpura
(G) Kawasaki syndrome
(H) Microscopic polyangiitis
(I) Polyarteritis nodosa
(J) Syphilitic aortitis
(K) Takayasu arteritis
(L) Thromboangiitis obliterans
(M) Wegener granulomatosis
Respuesta: M
The correct answer is M. Patients with Wegener granulomatosis are young (usually younger than 40 years) and have a history of chronic upper respiratory symptoms due to such conditions as otitis media, sinusitis, and pharyngitis. Later, the lungs (with hemoptysis and respiratory difficulties), as well as the kidneys (hematuria is the most frequent manifestation), become involved. ESR is elevated; fever, malaise, and weight loss are frequent. The presence of circulating autoantibodies of c-ANCA type supports a diagnosis of Wegener granulomatosis.
Aortic dissection (choice A) manifests with chest pain of sudden onset radiating to the back, signs of shock, and severe distress.
Atherosclerotic disease (choice B) would not explain any of these clinical presentations.
Churg-Strauss syndrome (choice C) is a vasculitis affecting small vessels of lungs, kidneys, skin, and other organs. It is usually associated with eosinophilia and p-ANCA antibodies.
Cryoglobulinemic vasculitis (choice D) results from deposition of immune complexes composed of immunoglobulins that precipitate at cold temperatures in capillaries and venules. The skin is frequently involved.
Giant cell arteritis (choice E) most commonly affects the branches of the external carotid artery, especially the temporal artery. It manifests in middle-aged or elderly persons with headache, fever, and malaise. It may lead to blindness if not promptly treated.
Henoch-Schönlein purpura (choice F) is associated with deposition of immune complexes containing IgA in the skin (purpura), gastrointestinal tract (abdominal pain and melena), joints (arthralgia), and glomeruli (hematuria).
Kawasaki syndrome (choice G) predominantly affects children. Cervical lymphadenopathy, inflammation of conjunctiva and oral mucous membranes, a desquamative rash typically involving the palms, and fever are the most typical manifestations. Vasculitis of coronary arteries occurs in 80% of cases and is the most common cause of death.
Microscopic polyangiitis (choice H) is a vasculitis similar to polyarteritis nodosa but results from an immune response to circulating agents, such as drugs (antibiotics) or infectious agents. Circulating p-ANCA are frequently found.
Polyarteritis nodosa (choice I) may affect medium-sized arteries of kidneys, heart, gastrointestinal tract, joints, muscles, and central and peripheral nervous systems. It is not associated with circulating ANCA.
Syphilitic aortitis (choice J) is a complication of the tertiary stage of syphilis. It results from endarteritis obliterans involving the vasa vasorum of the ascending aorta, which undergoes scarring and dilatation with consequent aortic regurgitation.
Thromboangiitis obliterans (choice L) occurs almost exclusively in young male smokers. It is an inflammatory condition affecting the neurovascular bundles in the limbs and causing ischemia, venous thrombosis, and pain due to nerve involvement.
A 2-year-old boy is brought by his parents to the physician because of an itchy rash that has been a persistent problem since 6 months of age. The rash is dry, erythematous, and scaly, and chiefly involves the face and dorsal surfaces of hands and feet. The lesions undergo alternating periods of remission and exacerbations. Local steroid treatment has provided temporary relief. Examination also reveals dry skin and pronounced skin markings on plantar and palmar surfaces. Family history is significant for allergic rhinitis and asthma in several relatives. The child’s development is otherwise normal.
(A) Allergic contact dermatitis
(B) Atopic dermatitis
(C) Hypothyroidism
(D) Immunodeficiency state
(E) Irritant contact dermatitis
(F) Langerhans cell histiocytosis
(G) Lupus erythematosus
(H) Nummular eczema
(I) Psoriasis
(J) Scabies
(K) Seborrheic dermatitis
(L) Tinea corporis
(M) Vitamin deficiency
Respuesta: B
The correct answer is B. The history of this case outlines the characteristic clinical features of one of the most common disorders of infants and young children, i.e., atopic dermatitis. Although its pathogenesis is not entirely clear, a pathologic predisposition to abnormal immune reactions plays a crucial role, which is emphasized by the frequent occurrence of allergic diseases in family members (e.g., allergic rhinitis, asthma, food intolerance). The manifestations of atopic dermatitis vary with age. Infants and children up to 2 years present with a scaly, erythematous, itchy rash distributed mainly on the face and dorsal hands and feet. Dry skin and accentuation of skin markings are additional frequent features. As the child grows, the distribution changes, involving predominantly flexural areas. The course is punctuated by periods of remission and exacerbation, but often the disease resolves spontaneously in late childhood. In a few unfortunate individuals, it persists into adulthood. Topical application of steroids affords temporary improvement, but cases of widespread skin involvement may require short-term systemic treatment. Systemic steroids, however, are associated with growth retardation.
Allergic contact dermatitis (choice A) is mediated by a type IV hypersensitivity reaction to a variety of allergens. Thousands of substances have been identified that can trigger this condition. Soaps, cosmetics, plastic compounds, glues, and fabrics are just a few examples of common allergens. The distribution of eczematous lesions and the clinical history are the most helpful elements for the diagnosis.
Hypothyroidism (choice C) should be considered as a possible cause of skin manifestations that may mimic eczematous reactions. Pretibial myxedema and loss of hair in the lateral eyebrows are the most typical cutaneous manifestations of hypothyroidism.
Immunodeficiency states (choice D) are associated with a wide range of cutaneous lesions, including chronic eczema and erythroderma. However, other signs of immune depression are usually present, including recurrent infections, diarrhea, and failure to thrive.
Irritant contact dermatitis (choice E) is caused by the irritating action of chemicals on the skin. It is not due to an allergic (i.e., immune-mediated) mechanism, but the skin manifestations are virtually indistinguishable from those of allergic contact dermatitis. Erythematous patches of itchy eczema develop in the areas in contact with the causative irritant. Again, distribution of lesions and clinical history are the most important clues to a correct diagnosis.
Langerhans cell histiocytosis (choice F) may manifest with three different syndromes. The systemic form (Letterer-Siwe disease) is the least frequent but most severe and affects infants and young children. It manifests with innumerable papules that may be mistaken for eczema. The child, however, appears acutely ill, with fever and severe systemic symptoms. The other forms affect older children or adults and manifest with localized formation of granuloma-like collections of Langerhans cells in skin, bone, and other organs.
Lupus erythematosus (choice G), both in its systemic form and skinlimited (discoid) variants, results in a classic erythematous rash that involves predominantly the face in a “butterfly-like” pattern. The lesions are exacerbated by exposure to sunlight.
Psoriasis (choice I) may be confused with seborrheic dermatitis since its characteristic silvery scaly lesions develop in the scalp as well. Indeed, there seems to be a “continuum” of clinical expressions encompassing seborrheic dermatitis, common dandruff, and psoriasis.
Scabies (choice J) is a disease due to Sarcoptes scabiei that manifests with a papulovesicular rash in a linear distribution. The preferred sites are inguinal, axillary, and finger webs. Linear burrows can be seen on close examination. The mite can be identified by applying a drop of mineral oil on suspected lesions, obtaining scrapings from these areas, and examining the scrapings under a magnifying lens or a 10-power objective.
Tinea corporis (choice L) is due to dermatophytes, fungal organisms that colonize the most superficial layers of the epidermis. The most characteristic appearance of skin lesions associated with tinea corporis is a ring-like pattern, due to central clearing as the infection spreads in a radial fashion. KOH-treated skin scrapings examined under a microscope allow identification of spores or hyphae.
A number of vitamin deficiencies (choice M) may cause skin alterations. Among these are deficiencies of vitamins A, B, C, and K
A 50-year-old man comes to his physician because of a 2-week history of pruritic rash on the extensor surfaces of legs and arms. The patient reports no family or personal history of allergic disorders. The rash is characterized by erythematous, crusted, coinshaped plaques. Skin scrapings treated with KOH do not reveal any spores or hyphae on microscopic examination. Physical examination is otherwise unremarkable.
(A) Allergic contact dermatitis
(B) Atopic dermatitis
(C) Hypothyroidism
(D) Immunodeficiency state
(E) Irritant contact dermatitis
(F) Langerhans cell histiocytosis
(G) Lupus erythematosus
(H) Nummular eczema
(I) Psoriasis
(J) Scabies
(K) Seborrheic dermatitis
(L) Tinea corporis
(M) Vitamin deficiency
Respuesta: H
The correct answer is H. As the name suggests, nummular eczema is characterized by coin-shaped lesions, which are covered by a uniform scaly crust. Itching may or may not manifest. It usually affects adult individuals without any apparent predisposing condition, except for a long-lasting tendency for dry skin. It is not caused by fungi, and microscopic examination of KOH-treated skin scrapings thus fails to reveal hyphae or spores. In contrast to fungal infections, the lesions of nummular eczema are homogeneous and do not show central clearing. The pathogenesis of nummular eczema is obscure and appears unrelated to allergic mechanisms. Fortunately, the lesions undergo spontaneous resolution in most of the cases without any need for therapy, except for symptomatic treatment if pruritus is present.
Allergic contact dermatitis (choice A) is mediated by a type IV hypersensitivity reaction to a variety of allergens. Thousands of substances have been identified that can trigger this condition. Soaps, cosmetics, plastic compounds, glues, and fabrics are just a few examples of common allergens. The distribution of eczematous lesions and the clinical history are the most helpful elements for the diagnosis.
Hypothyroidism (choice C) should be considered as a possible cause of skin manifestations that may mimic eczematous reactions. Pretibial myxedema and loss of hair in the lateral eyebrows are the most typical cutaneous manifestations of hypothyroidism.
Immunodeficiency states (choice D) are associated with a wide range of cutaneous lesions, including chronic eczema and erythroderma. However, other signs of immune depression are usually present, including recurrent infections, diarrhea, and failure to thrive.
Irritant contact dermatitis (choice E) is caused by the irritating action of chemicals on the skin. It is not due to an allergic (i.e., immune-mediated) mechanism, but the skin manifestations are virtually indistinguishable from those of allergic contact dermatitis. Erythematous patches of itchy eczema develop in the areas in contact with the causative irritant. Again, distribution of lesions and clinical history are the most important clues to a correct diagnosis.
Langerhans cell histiocytosis (choice F) may manifest with three different syndromes. The systemic form (Letterer-Siwe disease) is the least frequent but most severe and affects infants and young children. It manifests with innumerable papules that may be mistaken for eczema. The child, however, appears acutely ill, with fever and severe systemic symptoms. The other forms affect older children or adults and manifest with localized formation of granuloma-like collections of Langerhans cells in skin, bone, and other organs.
Lupus erythematosus (choice G), both in its systemic form and skinlimited (discoid) variants, results in a classic erythematous rash that involves predominantly the face in a “butterfly-like” pattern. The lesions are exacerbated by exposure to sunlight.
Psoriasis (choice I) may be confused with seborrheic dermatitis since its characteristic silvery scaly lesions develop in the scalp as well. Indeed, there seems to be a “continuum” of clinical expressions encompassing seborrheic dermatitis, common dandruff, and psoriasis.
Scabies (choice J) is a disease due to Sarcoptes scabiei that manifests with a papulovesicular rash in a linear distribution. The preferred sites are inguinal, axillary, and finger webs. Linear burrows can be seen on close examination. The mite can be identified by applying a drop of mineral oil on suspected lesions, obtaining scrapings from these areas, and examining the scrapings under a magnifying lens or a 10-power objective.
Tinea corporis (choice L) is due to dermatophytes, fungal organisms that colonize the most superficial layers of the epidermis. The most characteristic appearance of skin lesions associated with tinea corporis is a ring-like pattern, due to central clearing as the infection spreads in a radial fashion. KOH-treated skin scrapings examined under a microscope allow identification of spores or hyphae.
A number of vitamin deficiencies (choice M) may cause skin alterations. Among these are deficiencies of vitamins A, B, C, and K
A 35-year-old man with AIDS develops a scaling, oily erythema involving the scalp, eyebrows, nasolabial folds, back, chest, and umbilicus.
(A) Allergic contact dermatitis
(B) Atopic dermatitis
(C) Hypothyroidism
(D) Immunodeficiency state
(E) Irritant contact dermatitis
(F) Langerhans cell histiocytosis
(G) Lupus erythematosus
(H) Nummular eczema
(I) Psoriasis
(J) Scabies
(K) Seborrheic dermatitis
(L) Tinea corporis
(M) Vitamin deficiency
Respuesta: K
The correct answer is K. The most typical presentation of seborrheic dermatitis is a scaly, oily, dandruff-like rash on the scalp and eyebrows of infants (cradle cap), which is well known to all mothers. Adult patients with a congenital tendency for seborrhea may develop seborrheic dermatitis in a typical distribution, involving forehead, nasolabial folds, chin, beard area, chest, and anterior chest. Up to 50% of AIDS patients develop widespread seborrheic dermatitis, often in unusual locations. Other patients prone to this skin condition are those with Parkinson disease and those who are elderly and acutely ill.
Allergic contact dermatitis (choice A) is mediated by a type IV hypersensitivity reaction to a variety of allergens. Thousands of substances have been identified that can trigger this condition. Soaps, cosmetics, plastic compounds, glues, and fabrics are just a few examples of common allergens. The distribution of eczematous lesions and the clinical history are the most helpful elements for the diagnosis.
Hypothyroidism (choice C) should be considered as a possible cause of skin manifestations that may mimic eczematous reactions. Pretibial myxedema and loss of hair in the lateral eyebrows are the most typical cutaneous manifestations of hypothyroidism.
Immunodeficiency states (choice D) are associated with a wide range of cutaneous lesions, including chronic eczema and erythroderma. However, other signs of immune depression are usually present, including recurrent infections, diarrhea, and failure to thrive.
Irritant contact dermatitis (choice E) is caused by the irritating action of chemicals on the skin. It is not due to an allergic (i.e., immune-mediated) mechanism, but the skin manifestations are virtually indistinguishable from those of allergic contact dermatitis. Erythematous patches of itchy eczema develop in the areas in contact with the causative irritant. Again, distribution of lesions and clinical history are the most important clues to a correct diagnosis.
Langerhans cell histiocytosis (choice F) may manifest with three different syndromes. The systemic form (Letterer-Siwe disease) is the least frequent but most severe and affects infants and young children. It manifests with innumerable papules that may be mistaken for eczema. The child, however, appears acutely ill, with fever and severe systemic symptoms. The other forms affect older children or adults and manifest with localized formation of granuloma-like collections of Langerhans cells in skin, bone, and other organs.
Lupus erythematosus (choice G), both in its systemic form and skinlimited (discoid) variants, results in a classic erythematous rash that involves predominantly the face in a “butterfly-like” pattern. The lesions are exacerbated by exposure to sunlight.
Psoriasis (choice I) may be confused with seborrheic dermatitis since its characteristic silvery scaly lesions develop in the scalp as well. Indeed, there seems to be a “continuum” of clinical expressions encompassing seborrheic dermatitis, common dandruff, and psoriasis.
Scabies (choice J) is a disease due to Sarcoptes scabiei that manifests with a papulovesicular rash in a linear distribution. The preferred sites are inguinal, axillary, and finger webs. Linear burrows can be seen on close examination. The mite can be identified by applying a drop of mineral oil on suspected lesions, obtaining scrapings from these areas, and examining the scrapings under a magnifying lens or a 10-power objective.
Tinea corporis (choice L) is due to dermatophytes, fungal organisms that colonize the most superficial layers of the epidermis. The most characteristic appearance of skin lesions associated with tinea corporis is a ring-like pattern, due to central clearing as the infection spreads in a radial fashion. KOH-treated skin scrapings examined under a microscope allow identification of spores or hyphae.
A number of vitamin deficiencies (choice M) may cause skin alterations. Among these are deficiencies of vitamins A, B, C, and K
A 50-year-old man comes to the physician for a health maintenance examination. On this occasion, and on two subsequent visits, his blood pressure is found to be elevated. On the last visit, his blood pressure is 145/98 mm Hg. He denies any significant medical problems, but his family history is significant for high blood pressure, premature coronary artery disease, and diabetes mellitus. Physical examination reveals no abnormalities.
(A) Chest x-ray film
(B) Dexamethasone suppression test
(C) Growth hormone glucose suppression test
(D) Measurement of urinary catecholamines
(E) Plasma lipids
(F) Plasma renin level
(G) Thyroid function tests
(H) Two-dimensional echocardiogram
Respuesta: E
The correct answer is E. Uncomplicated cases of systemic hypertension are associated with few or no laboratory abnormalities. Some laboratory tests are recommended depending on the clinical history. If there is no clinical evidence of secondary hypertension, the following tests are generally considered satisfactory: fasting glucose to exclude diabetes; serum electrolytes to exclude hyperaldosteronism; creatinine and urinalysis to detect signs of renal impairment; plasma lipids (choice E) to identify additional risk factors for atherosclerosis; and ECG to identify signs of left ventricular hypertrophy (LVH). ECG is not very sensitive in detecting LVH, however, and some authors recommend performing M-mode echocardiography on all patients with newly diagnosed hypertension. However, the management would not be affected by the finding of mild degrees of LVH on echocardiography.
Chest x-ray films (choice A) are usually useless in diagnostic workup of hypertension.
The dexamethasone suppression test (choice B) is specific for hypercortisolism (Cushing disease).
A growth hormone (GH) glucose suppression test (choice C) is performed after an overnight fast to confirm/rule out acromegaly. Oral administration of 100 g glucose normally suppresses plasma levels of GH. This response is not observed in the presence of abnormal GH production.
Measurement of urinary catecholamines (choice D) is a highly specific test for pheochromocytoma, which manifests with attacks of headache, anxiety, palpitations, and other “sympathetic” symptoms. Hypertension may be paroxysmal or sustained. Thyroid function tests are usually normal, but hyperglycemia and glycosuria are often detected.
Plasma renin level (choice F) is an additional test to perform if there are clinical or laboratory signs of hyperaldosteronism. This condition (usually due to an isolated adrenal adenoma) is associated with hypokalemia and low plasma renin activity.
Two-dimensional echocardiogram (choice H) is too expensive for the initial diagnostic work-up of hypertension. It is definitely more sensitive than ECG for early detection of LVH, but no more so than M-mode echocardiography
A 30-year-old woman comes to the physician for a health maintenance examination. Her blood pressure is 145/65 mm Hg. Her pulse is irregular, with an average rate of 100/min. She admits having increasing anxiety, sleeplessness, and palpitations in the past 3 months. She also had a 4-kg (8.8 lb) weight loss over the same period. Examination reveals fine tremors of the hands, moist skin, and hyperreflexia.
(A) Chest x-ray film
(B) Dexamethasone suppression test
(C) Growth hormone glucose suppression test
(D) Measurement of urinary catecholamines
(E) Plasma lipids
(F) Plasma renin level
(G) Thyroid function tests
(H) Two-dimensional echocardiogram
Respuesta: G
The correct answer is G. In this case, the patient presents with hypertension associated with signs and symptoms highly suggestive of hyperthyroidism. Thus, more focused laboratory testing is warranted to confirm the underlying condition. Measurement of thyroid hormone and TSH (choice G) will allow a diagnosis of hyperthyroidism. ECG is necessary to check for the presence and type of arrhythmia. Hyperthyroidism often causes not only sinus tachycardia, but also atrial fibrillation. Note that the patient has systolic hypertension, but the diastolic pressure is low.
Chest x-ray films (choice A) are usually useless in diagnostic workup of hypertension.
The dexamethasone suppression test (choice B) is specific for hypercortisolism (Cushing disease).
A growth hormone (GH) glucose suppression test (choice C) is performed after an overnight fast to confirm/rule out acromegaly. Oral administration of 100 g glucose normally suppresses plasma levels of GH. This response is not observed in the presence of abnormal GH production.
Measurement of urinary catecholamines (choice D) is a highly specific test for pheochromocytoma, which manifests with attacks of headache, anxiety, palpitations, and other “sympathetic” symptoms. Hypertension may be paroxysmal or sustained. Thyroid function tests are usually normal, but hyperglycemia and glycosuria are often detected.
Plasma renin level (choice F) is an additional test to perform if there are clinical or laboratory signs of hyperaldosteronism. This condition (usually due to an isolated adrenal adenoma) is associated with hypokalemia and low plasma renin activity.
Two-dimensional echocardiogram (choice H) is too expensive for the initial diagnostic work-up of hypertension. It is definitely more sensitive than ECG for early detection of LVH, but no more so than M-mode echocardiography